You are on page 1of 67

2011 Surgery Section Part II

Multiple Choice
Identify the choice that best completes the statement or answers the question.

____ 1. What is the symptomatic term for hives?


a. Rash c. Urticaria
b. Eruption d. Dermatitis
____ 2. Melanin is found in what layer of the epidermis?
a. Epithelium c. Dermal
b. Squamous d. Basal
____ 3. What term relates to the connecting of the skin to the underlying muscles?
a. Dermis c. Hypodermis
b. Epidermis d. Sebaceous
____ 4. What term best describes a mass of hyperplastic scar tissue?
a. Keliod c. Dermatofibroma
b. Pilonidal Cyst d. Congenital nevus
____ 5. What is commonly known as a boil of the skin?
a. Abscess c. Carbuncle
b. Furuncle d. Impetigo
____ 6. What are the layers of the skin?
a. Epidermis and Dermis
b. Epidermis, Dermis and Fascia
c. Epidermis, Dermis, Subcutaneous Tissue and Fascia
d. Epidermis and Fascia
____ 7. What is the correct diagnostic code to report for treatment of a melanoma in-situ of the left arm?
a. 173.6 c. 172.6
b. 232.6 d. 238.2
____ 8. A patient is taken to surgery for the removal of a squamous cell carcinoma of the right thigh. Pathology
indicated that this carcinoma is a metastasis of a previous squamous cell carcinoma of the trunk. What is the
correct diagnostic code for today’s procedure?
a. 173.7 c. 198.5
b. 198.2 d. 173.5
____ 9. What would be the correct diagnostic code to report an open wound of the right leg related to a non-healing
operative wound of squamous cell carcinoma?
a. 173.7 c. 890.0
b. 998.59 d. V10.83
____ 10. A patient presented to the office with a suspicious lesion of the nose. The physician took a biopsy of the
lesion and pathology determined the lesion to be a dysplastic nevus. What is the correct diagnosis code to
report?
a. 173.3 c. 239.2
b. 216.3 d. 238.2
____ 11. Joe has a terrible problem with ingrown toenails. He goes to the podiatrist to have a nail removed along with
the nail matrix. What CPT® code(s) should be reported?
a. 11730 c. 11752
b. 11750 d. 11720
____ 12. What CPT® code(s) would best describe the treatment of 9 plantar warts removed and 6 flat warts all
destroyed with cryosurgery during the same office visit?
a. 17110, 17111-52 c. 17110, 17003
b. 17110 d. 17111
____ 13. What CPT® code(s) describe the destruction of 16 lesions using cryosurgery to the upper torso on a patient
diagnosed with molluscum contagiosum?
a. 17000, 17003 x 2 c. 17004
b. 17110, 17003 d. 17111
____ 14. While whittling a piece of wood, the patient sustained an avulsion injury to a portion of his left index finger
and underwent formation of a direct pedicle graft with transfer from his left middle finger. Immobilization
was accomplished with a plaster splint. What CPT® code(s) should be reported?
a. 15574 c. 15750
b. 15740 d. 15758
____ 15. A patient presents to the physician with multiple burns. After examination the physician determines the
patient has 3rd degree burns of the anterior portion of his left leg, below the knee extending to the foot
(4.5%). He also has 3rd degree burns of the anterior portion of the left side of his chest (4.5%). The patient
has 2nd degree burns of the posterior portion of his back and left arm (13.5%). What ICD-9-CM code(s)
should be reported?
a. 945.29, 945.19, 948.31
b. 942.34, 945.31, 948.31
c. 945.10, 945.13, 942.12, 943.30, 942.24, 948.31
d. 945.39, 942.32, 942.24, 943.20, 948.20
____ 16. A patient presents to the physician to discuss their acne and ask the physician about a suspicious lesion of
their left ear. The patient and physician discuss further treatment of the acne and agree to take a biopsy of the
lesion of the ear. Billing was sent prior to receiving the pathology report. What ICD-9-CM code(s) should be
reported?
a. 706.1, 238.2 c. 706.1, 239.2
b. 706.1 d. 706.1, 173.2
____ 17. The patient is here to see us in follow-up for a keloid that was excised from his neck in November of last
year. He believes that it’s coming back. He does have a recurrence of the keloid on the superior portion of
the scar. Since the keloid is still small, the options of an injection or radiation to the area were discussed.It
was agreed that our next course should be a Kenalog injection. The risks associated with the procedure were
discussed with the patient. Informed consent was obtained. The area was infiltrated with 1.5 cc of
medication. This was a mixture of 1 cc of 40-mg Kenalog and 0.5 cc of 1% lidocaine with epinephrine. He
tolerated the procedure well. What CPT® and ICD-9-CM code(s) should be reported?
a. 11900, 11901 x 7, J3301, 706.1 c. 11901, J3301, 701.4
b. 11900, J3301, 701.4 d. 11901 x 8, J3301, 238.2
____ 18. A patient presents with a recurrent seborrheic keratosis of the left cheek. The area was marked for a shave
removal. The area was infiltrated with local anesthetic, prepped and draped in a sterile fashion. The lesion
measuring 1.8 cm was shaved using an 11-blade. Meticulous hemostasis was achieved using light pressure.
The specimen was sent for permanent pathology. The patient tolerated the procedure well. What CPT®
code(s) should be reported?
a. 11200 c. 11442
b. 11312 d. 11642
____ 19. The patient has a suspicious lesion of the left jaw line. Clinical diagnosis of this lesion is unknown, but due
to the appearance malignancy is a realistic concern. The lesion was excised into the subcutaneous fat
measuring .8 cm and margins of .1 cm on each side. Hemostasis was achieved using light pressure. The
wound was closed in layers using 5.0 monocryl and 6.0 Prolene. Pathology revealed this lesion to be a benign
nevus with clear margins. What CPT® and ICD-9-CM code(s) should be reported?
a. 13131, 11441-51, 173.3 c. 12051, 11441-51, 216.3
b. 13131, 11441-51, 172.3 d. 12051, 11641-51, 216.3
____ 20. Patient has a suspicious lesion of the right axilla. The area was infiltrated with local anesthetic and prepped
and draped in a sterile fashion. With the use of a 3 mm punch tool the lesion was excised and closed with 5.0
Prolene suture. Pathology report indicated this was a seborrheic keratosis. What CPT® and ICD-9-CM
codes should be reported?
a. 11100, 702.19 c. 11400, 702.11
b. 11400, 702.19 d. 11100, 702.11
____ 21. The patient is here to see us about some skin tags of her neck and both underarms. She has had these lesions
for some time and they are irritated by her clothing, they itch and at times have burning sensation to them.
We discussed the treatments options along with the risks. Informed consent was obtained and we proceeded.
We removed 16 skin tags from the right axilla, 16 skin tags from the left axilla, 10 from the right neck and 17
from the left neck. What CPT® and ICD-9-CM codes should be reported?
a. 11057, 216.5, 216.4 c. 11200, 11201 x 4, 11201-52, 701.9
b. 11200, 216.5, 216.4 d. 11200, 11201 x 5, 701.9
____ 22. Patient presents to the physician for the removal of a squamous cell carcinoma of the right cheek. After the
area being prepped and draped in a sterile fashion the surgeon measured the lesion, documenting the size of
the lesion to be 2.3 cm in its’ largest diameter, additional the physician took margins of 2mm on each side of
the lesion. The patient tolerated the procedure well. What CPT® code(s) should be reported?
a. 11642, 12013 c. 11643, 12013
b. 11643 d. 11442
____ 23. Patient is diagnosed with actinic keratosis of the chest and arms, presents to her physician office for the
destruction of these lesions. Using cryosurgery the physician destroys 7 lesions on the right arm, 4 lesions on
the left forearm and 8 lesions on the chest. What CPT® and ICD-9-CM codes should be reported?
a. 17000, 17003 x 18, 239.2 c. 17004, 702.0
b. 17003 x 19, 238.2 d. 17000, 17003 x13, 17004, 702.0
____ 24. The patient is diagnosed with superficial basal cell carcinoma of the neck and cheek. After discussion with
the physician about different treatment options the patient decides to have these lesions destroyed using
cryosurgery. Consent is obtained and the areas are prepped in a sterile fashion. With the use of cryosurgery
the physician destroys the lesion on the neck measuring 2.3 cm and the lesion on the cheek measuring 0.8 cm.
What CPT® code(s) should be reported?
a. 17272, 17281-51 c. 17273, 17281-51, 11623-51, 11641-51
b. 17273, 17281-51 d. 11623, 11641-51
____ 25. Patient is a 69-year-old woman with a biopsy proved squamous cell carcinoma of her left forearm measuring
2.3 cm in greatest diameter. The area was marked with 4 mm gross normal margins. This area was removed
as drawn, and the surgeon then incised his planned rhomboid flap, elevating the full-thickness flap into the
defect and closing the sites in layers using 3-0 Monocryl, 4-0 Monocryl and 5-0 Prolene. The patient
tolerated the procedure well. Final measurements were 2.7 cm x 2.1 cm.
a. 14020 c. 13101, 11603-51
b. 14020, 11603-51 d. 15100, 11603-51
____ 26. A 50-year-old female has telangiectasias of the face on both cheeks. She is very bothered by this and presents
to have them destroyed via laser. The physician lasers 2 lesions on each cheek.
a. 17106 c. 17000, 17003
b. 17110 d. 17263
____ 27. Meredith has breast cancer on the left side, diagnosed with an excisional biopsy last week. Today she is
having a radical mastectomy, Urban type, and concurrently a single pedicle TRAM flap reconstruction with
supercharging. What CPT® code(s) should be reported?
a. 19367-LT, 19307-LT, 51-LT c. 19368-LT, 19306-51-LT
b. 19305-LT, 19367-51-LT, 19368-51-LT d. 19367-LT, 19302-51-LT
____ 28. A 14-year-old boy was thrown against the window of the car on impact. The resulting injury was a star shaped
pattern cut into the top of his head. On presentation to the ED, the MD on call for plastic surgery was asked to
evaluate the injury and repair it. The surgeon performed an expanded problem focused H&P. Medical
Decision Making was moderate. The total length of the intermediate repair was 5+ 4+ 4+ 5 cm. The star like
shape allowed the surgeon to pull the wound edges together nicely in a natural Y plasty in two spots. What
CPT® code(s) should be reported for the repair?
a. 14041 c. 13121
b. 14040 d. 12035
____ 29. A 63-year-old patient arrives for skin tag removal. As previously noted in her other visit, she has 3 located on
her face, 4 on her shoulder and 15 on her back. The physician removes all the skin tags with no complications.
What is the appropriate CPT® code(s) for this encounter?
a. 11201 c. 11200, 11201-52
b. 11201, 11201-51 d. 11200, 11201
____ 30. A 45-year-old male with a previous biopsy positive for malignant melanoma, presents for definitive excision
of the lesion. After induction of general anesthesia the patient is placed supine on the OR table, the left thigh
was prepped and draped in the usual sterile fashion. IV antibiotics are given as patient had previous MRSA
infection. The previous excisional biopsy site on the left knee had measured approximately 4 cm and was
widely elipsed with a 1.5 cm margin.. The excision was taken down to the underlying patellar fascia.
Hemostasis was achieved via electrocautery. The resulting defect was 11cm x 5cm. Wide advancement flaps
were created inferiorly and superiorly using electrocautery. This allowed the skin edges to come together
without tension. The wound was closed using interrupted 2-0 monocryl and 2 retention sutures were placed
using #1 Prolene. Skin was closed with a stapler.
What CPT® code(s) should be reported?
a. 27328 c. 14301, 27328-51
b. 14301 d. 15738, 11606-51
____ 31. Patient is an 81-year-old male with a biopsy proven basal cell carcinoma of this posterior neck just near his
hairline; additionally the patient had two additional areas of concern on his cheek. Informed consent was
obtained and the areas were prepped and draped in the usual sterile fashion. Attention was first directed to the
basal cell carcinoma of the neck, I excised the lesion measuring 2.6 cm as drawn down to the subcutaneous
fat. With extensive undermining of the wound I closed in layers using 4.0 monocryl, 5.0 Prolene and 6.0
Prolene; the wound measured 4.5cm. Attention was then directed to the other two suspicious lesions on his
cheek; after administering local anesthesia I proceeded to take a 3mm punch biopsy of each lesion and was
able to close with 5.0 Prolene. The patient tolerated the procedures well. Pathology later showed the basal
cell carcinoma was completely removed and the biopsies indicated actinic keratosis. What CPT® code(s)
should be reported?
a. 13132, 11623-51, 11100-59, 11101 c. 12042, 11623-51, 11100-59, 11101
b. 13131, 11622-51, 11100-59, 11100-59 d. 13132, 11623-51, 11440-51, 11440-51
____ 32. Patient is a 53-year-old female who yesterday underwent Mohs surgery with Dr. Smith to remove basal cell
carcinoma of her scalp. Due to the size of the defect Dr. Smith requested a Plastic Surgeon to reconstruct the
site. Dr. Jones discussed with the patient his planned closure which was a Ying-Yang type flap. The patient
agreed and we proceeded. The area was prepped and draped in a sterile fashion being careful to keep the
betadine solution out of the open wound. Wound preparation was done by excising an additional 1 mm
margin to freshen the wound and excising the wound deep sting superficial to the galea. Starting on the right
Dr. Jones incised his planned flap, elevating the flap with full-thickness and subcutaneous fat, staying superior
to the galea; then Dr. Jones incised his planned flap on the left elevating the flap with full-thickness and
subcutaneous fat. Both flaps were rotated together and the wound was temporarily closed using the skin
stapler. Once it was determined that there was minimal tension on the wound; the galea was approximated
using 4.0 Monocryl. The wound was then closed in layers using 5-0 Monocryl and a 35R skin stapler.
Meticulous hemostasis was achieved through-out the procedure with the Bovie cautery. Final measurements
of the wound were 36.25 cm squared. What CPT® code(s) should be reported?
a. 14021-22 c. 14301
b. 14021, 15004-51 d. 14301, 15004-51
____ 33. Patient presents to the emergency department with multiple lacerations due to a knife fight at the local bar.
After examination it was determined that these lacerations could be closed using local anesthesia. The areas
were prepped and draped in the usual sterile fashion. The surgeon documented the following closures; 7.6 cm
simple closure of the right forearm; 5.7 intermediate closure of the upper right arm; 6.5 simple closure of the
right wrist; 4.7 complex closure of the right neck; 10.3 intermediate closure of the upper chest; 8.9 cm
intermediate closure of the right abdominal area; 4.2 complex closure of the right ear and 3.9 intermediate
closure of the right cheek.
a. 13152, 13132, 12036, 12052, 12005
b. 13132, 13131, 12052, 12034, 12034, 12002, 12032, 12004
c. 13132, 13152, 12047
d. 13152, 13131, 12036, 12004
____ 34. Patient presents to the operative suite with a biopsy proven squamous cell carcinoma of the left ankle. A
decision was made to remove the lesion and apply a split thickness skin graft on the site. The lesion was
excised as drawn and documented as measuring 2.4 cm with margins. Using the Padgett dermatone the
surgeon harvested a split-thickness skin graft from the left thigh, which was meshed 1.5 x 1 and then inset
into the ankle wound using a skin stapler. Xeroform bolster was then placed of the skin graft using Xeroform
and 4-0 nylon and the lower extremity were wrapped with bulky cast padding and double Ace wrap. The skin
graft donor site was dressed with OpSite. The surgeon noted the skin graft measured 4.0 cm squared in total.
a. 15100, 11603-51, 173.7 c. 15120, 13100-51, 216.7
b. 15100, 173.7 d. 15240, 11603-51, 173.7
____ 35. Patient presents with a suspicious lesion on her left arm. With the patient’s permission the physician marked
the area for excision. The lesion measured 0.9 cm. The wound measuring 1.2 cm was closed in layers using
4-0 Monocryl and 5-0 Prolene. Pathology later reported the lesion to be a sebaceous cyst. What CPT®
code(s) should be reported?
a. 11401, 216.6 c. 13121, 11401-51, 216.6
b. 12031, 11401-51, 706.2 d. 11402, 706.2
____ 36. Operative Report:
Pre-Operative Diagnoses: Basal Cell Carcinoma, forehead
Basal Cell Carcinoma, right cheek
Suspicious lesion , left nose
Suspicious lesion, left forehead
Post-Operative Diagnoses: Basal Cell Carcinoma, forehead with clear margins
Basal Cell Carcinoma, right cheek with clear margins
Compound nevus, left nose with clear margins
Epidermal nevus, left forehead with clear margins

INDICATIONS FOR SURGERY: The patient is a 47-year-old white man with a biopsy-proven basal cell
carcinoma of his forehead and a biopsy-proven basal cell carcinoma of his right cheek. We were not quite sure
of the patient’s location of the basal cell carcinoma of the forehead whether it was a midline lesion or lesion to
the left. We felt stronger about the midline lesion, so we marked the area for elliptical excision in relaxed skin
tension lines of his forehead with gross normal margins of 1-2 mm and I marked the lesion of the left
forehead for biopsy. He also had lesion of his left alar crease that we marked for biopsy and then a large basal
cell carcinoma of his right cheek, which was more obvious. This was marked for elliptical excision with gross
normal margins of 2-3 mm in the relaxed skin tension lines of his face. I also drew a possible rhomboid flap
that we would use if the wound became larger. He observed all these margins in the mirror, so he could
understand the surgery and agree on the locations, and we proceeded.

DESCRIPTION OF PROCEDURE: All four areas were infiltrated with local anesthetic. The face prepped and
draped in sterile fashion. I excised the lesion of the forehead measuring 6-mm and right cheek measuring 1.3
cm as I had drawn them and sent in for frozen section. The biopsies were taken of the left forehead and left
nose using a 2-mm punch, and these wounds were closed with 6-0 Prolene. Meticulous hemostasis was
achieved of those wounds using Bovie cautery. I closed the cheek wound first. Defects were created at each
end of the wound to facilitate primary closure and because of this I considered a complex repair and the
wound was closed in layers using 4-0 Monocryl, 5-0 Monocryl and 6-0 Prolene, with total measurement of
2.1 cm. The forehead wound was closed in layers using 5-0 Monocryl and 6-0 Prolene, with total
measurement of 1.0 cm. Loupe magnification was used and the patient tolerated the procedure well.

What CPT® and ICD-9-CM codes should be reported?


a. 13121, 12051-51, 11642-51, 11640-51, 11100-58, 11100, 173.3, 232.3, 238.2, 216.3
b. 13131, 12051-51, 11642-51, 11641-51, 11100-59, 11101, 173.3, 216.3
c. 13131, 12052-51, 11442-51, 11440-51, 11100-59, 11101, 173.2, 173.4, 216.2, 216.3
d. 12053, 11643-51, 11100-58, 11101, 172.3, 173.3, 238.2, 239.2
____ 37. Operative Report
Pre-Operative and Post-Operative Diagnosis: Squamous cell carcinoma, left leg
Open wound, right leg
Personal history of squamous cell carcinoma, right leg

INDICATIONS FOR SURGERY: The patient is an 81-year-old white man with a biopsy-proven squamous
cell carcinoma of his left leg. I marked the areas for excision with gross normal margins of 5 mm, and I drew
my planned skin graft donor site from his left lateral thigh. He also had an open wound of his right leg from a
squamous cell carcinoma I had excised four months ago and the skin graft had not taken. So we plan on re-
skin grafting that area. The patient is aware of all of these markings, and he understood the surgery and the
location, and we proceeded.
DESCRIPTION OF PROCEDURE: The patient was taken to the operating room. IV Ancef was given. I used
plain lidocaine for his local anesthetic throughout the procedure until the skin grafts were inset. The anterior
of his leg and the thigh were infiltrated with local anesthetic. Both upper extremities were prepped and draped
circumferentially, which included the left thigh on the left side. I excised the lesion of his left leg as drawn
into the subcutaneous fat. Hemostasis achieved with the Bovie cautery. I then excised the wound of his right
leg to lower the bacterial counts. I took a 1-2 mm margin around the wound and excised the granulation tissue
as well. Hemostasis was achieved using the Bovie cautery. I then changed gloves. A split-thickness skin graft
was harvested from the left thigh using the Zimmer dermatome. This was meshed one and a half times one.
By this time, the pathology had returned showing that the margins were clear.
The skin grafts were inset on each leg wound using the skin stapler. Xeroform and gauze bolster was placed
over the skin graft using 4-0 nylon. The skin graft donor site was dressed with OpSite. The legs were further
dressed with heavy cast padding and the double Ace wrap. The patient tolerated the procedure well.

PROCEDURES: Excision squamous cell carcinoma, left leg with excised diameter of 2.5 cm, repaired with a
split-thickness skin graft measuring 5.1 cm squared. Excisional preparation of right leg wound repaired with
a split-thickness skin graft measuring 3.2 cm squared.

What CPT® and ICD-9-CM codes should be reported?


a. 15100, 11603, 15002, 173.7, 996.52, V10.83
b. 15100-RT, 15100-LT, 11603, 15002, 173.7, 996.52, V10.83
c. 15100-LT, 11403, 15100-RT, 173.7, V10.82
d. 15100-50, 11603, 890.0, 173.7, 996.52
____ 38. The patient is seen in follow-up for the excision of the basal cell carcinoma of his nose. I examined his nose
noting the wound has healed well. His pathology showed that the margins were clear. He has a mass of his
forehead that he says is from a piece of sheet metal that injured his forehead. He has an x-ray print that shows
a foreign body, so we have offered to remove that. After obtaining consent we proceeded. The area was
infiltrated with a local anesthetic. I had drawn for him how I would incise over the foreign body. He
observed this in the mirror so he could understand the surgery and agree on the location. I incised a thin
ellipse over the mass to give me better access to it and then the mass was removed. There was a capsule
around this, what appeared to be a black-colored metal that was stained, and I felt that could potentially cause
a permanent black mark on his forehead, so I offered to excise that and he wanted me to do so, so I went
ahead and removed the capsule with the stain and that removed all the black stain that I could see, because of
this I consider this to be a complicated procedure. Hemostasis was achieved with light pressure. The wound
was closed in layers using 4-0 Monocryl and 6-0 Prolene.

What CPT® and ICD-9-CM codes should be reported?


a. 10121, 709.4, V90.10 c. 10121, 729.6, V90.10
b. 11010, 709.4, V90.10 d. 11010, 729.6, V90.10
____ 39. The patient is here because the cyst of her chest has sort of come to a head and is still painful even though she
has been on antibiotics for a week. I offered to drain that for her. After obtaining consent, we infiltrated the
area with 1 cc of 1% lidocaine with epinephrine, prepped the area with Betadine and opened the cyst in the
relaxed skin tension lines of her chest, and removed the cystic material. There was no obvious purulence. We
are going to have her clean this with a little Q-tip. We will let that heal on its own and eventually excise it. I
will have her come back a week from Tuesday and we are going to reschedule her surgery. What CPT® and
ICD-9-CM codes should be reported?
a. 10040, 706.1 c. 10061, 706.2
b. 10060, 706.2 d. 10160, 996.54
____ 40. The patient is here in follow-up for the left capsulectomy. She had developed a seroma. It has been present
for about a week. It has been about the same volume. It does not hurt her any. She has not had any fever.
She has not had any trouble breathing.

PHYSICAL EXAMINATION: On examination she does have a seroma present and I have offered to aspirate
the area. I have told her that we usually try this for a few times and then if it has not resolved, usually we
give up and let it resolve on its own. One of the risks in doing this is pneumothorax, so we do not want to
push it too hard. We prepped her left chest with Betadine and with a 16-gauge needle sterilely aspirated 60 cc
of serosanguinous fluid. It was not cloudy at all. She had no trouble with her procedure and no difficulty
breathing. We are going to see her next week. She has asked if she can go ahead and be fitted for a prosthesis
and I am going to give her a prescription for one as long as she does not have the seroma when she goes to get
fitted I think it is fine. She is going to try to get a Spandex-type top to get some compression to the area.
What CPT® and ICD-9-CM codes should be reported for the procedure?
a. 10160-78, 998.13 c. 10140-78, 906.3
b. 10180-58, 998.12 d. 10140-58, 729.91
____ 41. Operative Report
PREOPERATIVE DIAGNOSIS: Basosquamous cell carcinoma, scalp.
POSTOPERATIVE DIAGNOSIS: Basosquamous cell carcinoma, scalp.
PROCEDURE PERFORMED: Excision of basosquamous cell carcinoma, scalp with Yin-Yang flap repair

ANESTHESIA: Local, using 4 cc of 1% lidocaine with epinephrine.


COMPLICATIONS: None.
ESTIMATED BLOOD LOSS: Less than 5 cc.
SPECIMENS: Basosquamous cell carcinoma, scalp sutured at 12 o’clock, anterior tip

INDICATIONS FOR SURGERY: The patient is a 43-year-old white man with a biopsy-proven basosquamous
cell carcinoma of his scalp measuring 2.1 cm. I marked the area for excision with gross normal margins of 4
mm and I drew my planned Yin-Yang flap closure. The patient observed these markings in two mirrors, so he
can understand the surgery and agreed on the location and we proceeded.
DESCRIPTION OF PROCEDURE: The area was infiltrated with local anesthetic. The patient was placed
prone, his scalp and face were prepped and draped in sterile fashion. I excised the lesion as drawn to include
the galea. Hemostasis was achieved with the Bovie cautery. Pathologic analysis showed the margins to be
clear. I incised the Yin-Yang flaps and elevated them with the underlying galea. Hemostasis was achieved in
the donor site using Bovie cautery. The flap rotated into the defect with total measurements of 2.9 cm x 3.2
cm. The donor sites were closed and the flaps inset in layers using 4-0 Monocryl and the skin stapler. Loupe
magnification was used. The patient tolerated the procedure well.

What CPT® and ICD-9-CM codes should be reported?


a. 14060, 172.3 c. 14041, 172.4
b. 14040, 173.4 d. 14020, 173.4
____ 42. The patient is coming in for removal of fatty tissue of the posterior iliac crest, abdomen, and the medial lateral
thighs. Suction-assisted lipectomy was then undertaken in the left posterior iliac crest area and this was
continued on the right and then the lateral trochanteric and posterior aspect of the medial thighs. The medial
thighs were suctioned followed by the abdomen. The total amount infused was 2300 cc and the total amount
removed was 2400 cc. The incisions were closed and a compression garment was applied. What CPT®
code(s) should be reported?
a. 15830, 15832 c. 15830, 15839, 15847
b. 15877, 15879 d. 15877, 15878
____ 43. Operative Report
PREOPERATIVE DIAGNOSIS: Diabetic foot ulceration.
POSTOPERATIVE DIAGNOSIS: Diabetic foot ulceration.
OPERATION PERFORMED: Debridement and split thickness autografting of left foot

ANESTHESIA: General endotracheal.

INDICATIONS FOR PROCEDURE: This patient with multiple complications from Type II diabetes has
developed ulcerations which were debrided and homografted last week. The homograft is taking quite nicely,
the wounds appear to be fairly clean; he is ready for autografting.

DESCRIPTION OF PROCEDURE: After informed consent the patient is brought to the operating room and
placed in the supine position on the operating table. Anesthetic monitoring was instituted, internal anesthesia
was induced. The left lower extremity is prepped and draped in a sterile fashion. Staples were removed and
the homograft was debrided from the surface of the wounds. One wound appeared to have healed; the
remaining two appeared to be relatively clean. We debrided this sharply with good bleeding in all areas.
Hemostasis was achieved with pressure, Bovie cautery, and warm saline soaked sponges. With good
hemostasis a donor site was then obtained on the left anterior thigh, measuring less than 100 cm 2. The wounds
were then grafted with a split-thickness autograft that was harvested with a patch of Brown dermatome set at
12,000 of an inch thick. This was meshed 1.5:1. The donor site was infiltrated with bupivacaine and dressed.
The skin graft was then applied over the wound, measured approximately 60 cm 2 in dimension on the left
foot. This was secured into place with skin staples and was then dressed with Acticoat 18's, Kerlix
incorporating a catheter, and gel pad. The patient tolerated the procedure well. The right foot was redressed
with skin lubricant sterile gauze and Ace wrap. Anesthesia was reversed. The patient was brought back to the
ICU in satisfactory condition.

What CPT® and ICD-9-CM codes should be reported?


a. 15220-58, 15004-58, 707.15, 250.80
b. 15120-58, 15004-58, 250.80, 707.15
c. 15950-78, 15004-78, 250.00, 707.14
d. 11044-78, 15120-78, 15004-78, 250.80, 707.15
____ 44. Operative Report
Diagnosis: Basal Cell Carcinoma
Procedure: Mohs micrographic excision of skin cancer.
Site: face left lateral canthus eyelid
Pre-operative size: 0.8 cm
Indications for surgery: Area of high recurrence, Area of functional and/or cosmetic importance Discussed
procedure including alternative therapy, expectations, complications, and the possibility of a larger or deeper
defect than expected requiring significant reconstruction. Patient’s questions were answered.

Local anesthesia 1:1 marcaine and 1% lidocaine with epinephrine. Sterile prep and drape.

Stage 1: The clinically apparent lesion was marked out with a small rim of normal appearing tissue and
excised down to subcutaneous fat level with a defect size of 1.2 cm. Hemostasis obtained and a pressure
bandage placed. The tissue was sent for slide preparation. Review of the slides show clear margins for the
site.

Repair: Complex repair.


Repair of Mohs micrographic surgical defect. Wound margins were extensively undermined in order to
mobilize tissue for closure. Hemostasis was achieved. Repair length 3.4 cm. Narrative: Burrows triangles
removed anteriorly (medial) and posteriorly (lateral). A layered closure was performed. Multiple buried
absorable sutures were placed to reappose deep fat. The epidermis and dermis were reapposed using
monofilament sutures. There were no complications; the patient tolerated the procedure well. Post-procedure
expectations (including discomfort management), wound care and activity restrictions were reviewed. Written
Instructions with urgent contact numbers given, Follow-up visit and suture removal in 3-5 days

What CPT® and ICD-9-CM codes should be reported?


a. 13152, 11642-51, 173.3 c. 17313, 13152-51, 173.1
b. 17311, 17312, 13152-51, 173.3 d. 17311, 13152-51, 173.1
____ 45. PREOPERATIVE & POSTOPERATIAVE DIAGNOSES:
1. Macromastia.
2. Back pain.
3. Neck pain.
4. Shoulder pain.
5. Shoulder grooving
6. Intertrigo.

NAME OF PROCEDURE:
1. Right breast reduction of 1950 g.
2. Right free-nipple graft.
3. Left breast reduction of 1915 g.
4. Left free-nipple graft.

INDICATIONS FOR SURGERY: The patient is a 43-year-old female with macromastia and associated back
pain, neck pain, shoulder pain, shoulder grooving and intertrigo. She desired a breast reduction. Because of
the
extreme ptotic nature of her breasts, we felt that she would need a free-nipple graft technique. In the
preoperative holding area, we marked her for this free-nipple graft technique of breast reduction. The patient
observed these markings so she could understand the surgery and agree on the location, and we proceeded.
The patient also was morbidly obese with a body mass index of 54. Because of this, we felt that she met the
criteria for DVT prophylaxis, which included Lovenox injection. The patient understood that this would
increase her risk of bleeding. She also made it known that she is a Jehovah's Witness and refused blood
products, but she did understand that her risk of bleeding would significantly increase and we proceeded.
DESCRIPTION OF PROCEDURE: The patient was given 40 mg of subcu Lovenox in the preoperative
holding area. She was then taken to the operating room. Bilateral thigh-high TED hose, in addition to bilateral
pneumatic compression stockings throughout the procedure. IV Ancef 1 g was given. Anesthesia was induced.
Both arms were secured on padded arm boards using Kerlix rolls. A similar body bear hugger was placed. The
chest and abdomen were prepped and draped in sterile fashion. I began by circumscribing around each nipple-
areolar complex using a 42-mm areolar marker. On each side the free-nipple grafts were harvested. They were
marked to be side specific and they were stored on the back table in moistened lap sponges. Meticulous
hemostasis was achieved using Bovie cautery. The tail of the apex of each breast was then deepithelialized
using the scalpel. I then amputated the inferior portion of the breast from the right side. Again, meticulous
hemostasis was achieved using the Bovie cautery. There were also large feeder vessels that were divided and
ligated using either a medium Ligaclip or 3-0 silk tie sutures. I then moved to the left and again amputated the
inferior portion of the breast. Meticulous hemostasis was achieved using the Bovie cautery. Each of these
wounds were then temporarily closed using the skin stapler. The patient was then sat up. I felt that we had
achieved a very symmetrical result. The new positions for the nipple-areolar complexes was marked with a
42-mm areolar marker and methylene blue. The patient was then placed in the supine position and the new
positions for the nipple-areolar complexes were deepithelialized using the scalpel. Meticulous hemostasis was
then achieved again using the Bovie cautery. The free-nipple grafts were then retrieved from the back table.
They were each defatted using scissors and they were placed in an onlay fashion on the appropriate side, and
each were inset using 5-0 plain sutures. Vents were made in the skin graft to allow for the egress of fluid on
each side, and then a vertical mattress suture was used tied over a piece of Xeroform in critical areas of each
of the nipple-areolar complexes. A Xeroform bolster wrapped over a mineral oil-moistened sponge
was affixed to each of the nipple-areolar complexes using 5-0 nylon suture. The vertical and transverse
incisions were then closed using 3-0 Monocryl, both interrupted and running suture, and 5-0 Prolene. The
patient tolerated the procedure well. Again, meticulous hemostasis was achieved using the Bovie cautery. She
was then given another 1 g of Ancef at the
2-hour mark by our anesthesiologist, and she was taken to the recovery room in good condition.

What CPT® code(s) should be reported?


a. 19316-50, 19355-59, 50 c. 19318-50, 19355-59, 50
b. 19318-50, 19350-59, 50 d. 19340-50, 19350-59, 50
____ 46. Operative Report
PREOPERATIVE DIAGNOSIS: Congenital left breast deformity.
POSTOPERATIVE DIAGNOSIS: Congenital left breast deformity.

PROCEDURE PERFORMED: Placement of left breast implant using mentor catalog #, lot #, serial #, 425 cc
smooth round moderate profile implant filled with 475 cc of normal saline for breast reconstruction.

INDICATIONS FOR SURGERY: The patient is a 34-year-old female who approximately 15 to 16 years ago
had a left breast implant placed for breast reconstruction for her congenitally underdeveloped left breast. This
implant ruptured and in late September 2008, I performed a capsulectomy and exchanged her ruptured
implant for a new implant. About a week after surgery, the patient developed an infection and because of that
infection, her implant had to be removed. The patient’s infection has completely resolved and she is now
ready to have her implant replaced. In the preoperative holding area, I marked her for the ideal position of
this implant and performed a breast exam that did not show that there was no mass in either breast and no
mass in axillae and we proceeded. We did discuss with the patient that even though her original implant was
placed in subglandular position I felt that it would be beneficial to place the implant behind her pectoralis
major muscle that is in submuscular position today and the patient agreed on this and we proceeded.
DESCRIPTION OF PROCEDURE: The patient was given 1 g of IV vancomycin. The patient was taken to
the operating room; general anesthesia was induced bilateral pneumatic compression stockings were worn
throughout the procedure. A lower body Bair Hugger was placed. Both arms were secured to padded
armboard using Kerlix roll. The neck, chest, axillae, and upper abdomen were prepped and draped in sterile
fashion. I began by incising the central portion of her previous scar. I then dissected down to the pectoralis
major muscle. A submuscular plane was developed through a lateral approach and the inferior and medial
origin of the muscle was partially divided using the Bovie cautery. Meticulous hemostasis was achieved
using the Bovie cautery. There were no signs of infection nor were there any pockets of seroma fluid or
hematoma. The wound was carefully inspected. Meticulous hemostasis had been achieved. Gloves were
then changed. The implant was opened. The air was evacuated. It was placed in the submuscular pocket and
the wound was temporarily closed using a skin stapler. The implant was then filled to its maximum volume
and that was 475 cc of normal saline. The patient was then sat up. I then adjusted the volume and ultimately
did feel that she needed a 475 cc for the breast symmetry with her contralateral breast. Once I was satisfied
with the position of the implant, the patient was then placed supine. Gloves were changed again. The fill
tube was removed and I then secured the filled valves digitally and the deepest layer of breast tissue was
closed using 3-0 Vicryl in running suture and the skin was closed in three layers using 4-0 Monocryl, 5-0
Monocryl, and 5-0 Prolene. The wound was dressed with Xeroform and gauze. The patient tolerated the
procedure well. She was taken to recovery room in good condition.

What CPT® and ICD-9-CM codes should be reported?


a. 19325-LT, 611.83 c. 19316-LT, 611.83
b. 19340-LT, 757.6 d. 19342-LT, 757.6
____ 47. INDICATIONS FOR SURGERY: The patient is an 82-year-old white man with a biopsy-proven basal cell
carcinoma of his right lower eyelid and cheek laterally. I marked the area for rhomboidal excision and I drew
my planned rhomboid flap. The patient observed these markings in a mirror, so he can understand the surgery
and agreed on the location and we proceeded.
DESCRIPTION OF PROCEDURE: The area was infiltrated with local anesthetic. The face prepped and
draped in sterile fashion. I excised the lesion as drawn into the subcutaneous fat. Hemostasis was achieved
using the Bovie cautery. Modified Mohs analysis showed the margin to be clear. I incised the rhomboid flap
as it was drawn and elevated the flap with a full-thickness of subcutaneous fat. Hemostasis was achieved in
the donor site and actually the Bovie cautery was not used, the handheld cautery was used. The flap was
rotated into the defect. The donor site was closed and flap inset in layers using 5-0 Monocryl and 6-0 Prolene.
The patient tolerated the procedure well. The total site measured 1.3 cm x 2.7 cm

What CPT® code(s) should be reported?


a. 14020 c. 14040, 14060
b. 14060 d. 11643
____ 48. A wire placement in the lower outer aspect of the right breast was placed by a radiologist the day prior to this
procedure. During this operative session, the surgeon created an incision through the wire track and the wire
track was followed down to its entrance into breast tissue. A nodule of breast tissue was noted immediately
adjacent to the wire. This entire area was excised by sharp dissection, sent to pathology and returned as a
benign lesion. Bleeders were cauterized, and subcutaneous tissues were closed with 3-0 vicryl. Skin edges
were approximated with 4-0 subcuticular sutures and adhesive strips were applied. The patient left the
operating room in satisfactory condition. What is the correct code for the surgeon’s services?
a. 19125 c. 19125, 19295
b. 11400 d. 19120
____ 49. A malignant lesion of the forehead measuring 1.0 cm was removed. The operative report states that skin
margins are 1.1 cm on all sides. Layered closure was performed. How is this coded?
a. 11644, 12052-51 c. 11604
b. 11602, 12052-51 d. 11602, 12051-51
____ 50. A 56-year-old pro golfer is having Mohs micrographic surgery for skin cancer on his forehead. The surgeon
takes him back for two sessions. The first session has 4 tissue blocks and the second has 6 tissue blocks, 2 of
which have toluidine blue. What is the best way to code for both sessions' service?
a. 17311, 17314, 17315, 88314 c. 17311, 17312, 17315
b. 17313, 17314, 17315 d. 17311, 17312, 17315, 88314
____ 51. A patient is seen in the same day surgery unit for an arthroscopy to remove some loose bodies in the shoulder
area. What CPT® code(s) should be reported?
a. 29805 c. 29807
b. 29806 d. 29819
____ 52. What hallux valgus surgery includes a tendon transplant?
a. Joplin procedure c. Mitchell procedure
b. Mayo procedure d. Keller procedure
____ 53. What is the acromion?
a. Part of the elbow joint c. Tendon in the shoulder
b. Ligament near the knee d. Extension of the scapula
____ 54. Hallux rigidus is condition affecting what part of the body?
a. Spine c. Foot
b. Ankle d. Knee
____ 55. What is segmental instrumentation?
a. Rods attached only at the top and bottom of a spinal fixation device.
b. Bone plates in more than one area of the skull
c. A spinal fixation device attached in at least three places
d. The instruments used in surgery of the metatarsals.
____ 56. A patient has a greenstick fracture of the arm. It is treated by surgically placing a bone plate on the distal
radial shaft. What ICD-9-CM code(s) should be reported?
a. 813.81 c. 813.21
b. 813.31 d. 733.12
____ 57. In the ICD-9-CM, what would you look for in the alphabetic index, to code a tear of the supraspinatus muscle
of the shoulder?
a. Rotator cuff, sprain c. Injury, shoulder
b. Sprain, shoulder d. Tear, rotator cuff
____ 58. A patient presents to the ED with back pain and is diagnosed with a lumbar sprain. What ICD-9-CM code(s)
should be reported?
a. 847.2 c. 724.6
b. 846.0 d. 724.2
____ 59. A patient complains of pain in his foot and heel, and is diagnosed with plantar fasciitis in the sole of his left
foot. What ICD-9-CM code(s) should be reported?
a. 729.5 c. 825.20
b. 728.71 d. 581.3
____ 60. The patient fell on location and fractured his femoral shaft in three places. He has to have an ORIF of the left
femur with an intramedullary nail and interlocking screws. The orthopedist also places the leg in a plaster
splint prior to leaving the OR. What CPT® code(s) should be reported?
a. 27245 c. 27513
b. 27507 d. 27506
____ 61. A 44-year-old male with biplanar deformity, acquired limb length discrepancies and tibial nonunion has
undergone deformity correction. He now requires exchange of a strut 45 days postoperatively. The
intraoperative mounting parameters, deformity parameters, and initial strut settings are inserted into the
computer prior to Jim’s discharge and a daily schedule is generated for him to perform the gradual deformity
correction necessary. What CPT® code(s) should be reported?
a. 20696 c. 99024
b. 20697 d. 20692, 20697
____ 62. A patient is given Xylocaine, a local anesthetic, by injection on the thigh above the site to be biopsied. A
small bore needle is then introduced into the muscle, about 3 inches deep, and a muscle biopsy is taken. What
CPT® code(s) should be reported for this service?
a. 20205 c. 20225
b. 20206 d. 27324
____ 63. The patient presents today for closed reduction of the nasal fracture. The depressed right nasal bone was
elevated using heavy reduction forceps while the left nasal bone was pushed to the midline. This resulted in
good alignment of the external nasal dorsum. What CPT® code(s) should be reported for this procedure?
a. 21325 c. 21315
b. 21310 d. 21337
____ 64. A 22-year-old female has a retained Kirschner wire in the left little finger. Using local anesthesia, the left
upper extremity was thoroughly cleansed with Betadine. The end portion of the little finger was opened by a
transverse incision through the subcutaneous tissue to the bone. The retained Kirschner wire was located
within the distal phalanx. It was removed and closed with sutures. What CPT® code(s) should be reported?
a. 10120-F4 c. 20670-F4
b. 20680-F4 d. 10121-F4
____ 65. A 63-year-old man presents with a neck mass to be excised. The neck mass was palpated and an incision was
then made and carried down through the dermis with electrocautery. The subcutaneous tissue of the skin was
opened encountering an organized mass that has a benign appearance of a lipoma. Using careful blunt and
sharp dissection, the mass measuring 5 cm was completely excised around its entire circumference leaving the
capsule intact. The mass was then removed from its posterior attachments using electrocautery. What CPT®
code(s) should be used for this procedure?
a. 11426 c. 11626
b. 21552 d. 21555
____ 66. A 27-year-old presents with right-sided thoracic myofascial pain. A 25 gauge 1.5 inch needle on a 10 cc
controlled syringe with 0.25% bupivacaine was used. After negative aspiration, 2 cc were injected into each
point. A total of four points were injected. A total of 8 cc of bupivacaine was used on the rhomboid major,
rhomboid minor, and levator scapular muscles. What CPT® code(s) should be used for this procedure?
a. 20550 x 4 c. 20552
b. 20553 d. 20552, 20553-51
____ 67. A 42-year-old with chronic right trochanteric bursitis is scheduled to receive an injection at the Pain Clinic. A
22-gauge spinal needle is introduced into the trochanteric bursa, and a total volume of 8 cc of normal saline
and 40 mg of Kenalog was injected. What CPT® code(s) should be reported?
a. 20610-RT c. 20550-RT
b. 27093-RT d. 20605-RT
____ 68. A 63-year-old man sustained a gunshot wound through the right maxillary sinus that penetrated through the
right neck. CT scan revealed no hard evidence of arterial injury but a bullet was directly in line with the
internal jugular vein. He was sent to the operating room for neck exploration to rule out vascular injury and
injury to the aerodigestive tract. A sternocleidomastoid incision was performed and carried down through the
platysma muscle. There was no penetration of the internal jugular vein, but a foreign body was identified
resting on the internal jugular vein at approximately the level of the angle of the mandible and removed. The
parotid gland was noted to have a blast injury near the tail. This was not surgically repaired or resected. Once
all bleeding was controlled, a 10 French round drain was placed in the wound. The wound was copiously
irrigated. The platysma muscle was closed and the skin was closed with subcuticular closure. What CPT®
code(s) should be reported?
a. 20525 c. 20100
b. 35201 d. 21899
____ 69. A 66-year-old has sustained a left proximal humerus fracture. Standard deltopectoral approach was used and
dissection was carried out down to the fracture site. The fracture site was identified and fragments were
mobilized and the humeral head fragments removed. Once this was done, the stem was prepared up to a size
10. A trial reduction was carried out with the DePuy trial stem and implant head. Sutures were placed in key
positions for closure of the tuberosities down to the shaft including sutures through the shaft. The shaft was
then prepared and cement was injected into the shaft. The implant was placed. Once the cement was
hardened, the head was placed on Morse taper and then reduced. A bone graft was placed around the area
where the tuberosities were being brought down. The tuberosities were then tied down with a suture
previously positioned. This gave excellent closure and coverage of the significant motion at the repair sites.
The wound was thoroughly irrigated. The skin was closed with Vicryl over a drain and also staples in the
epidermis. A sterile dressing and sling was applied. The patient was taken to recovery in stable condition. No
immediate complications. What CPT® code(s) should be reported?
a. 23616-LT c. 23615-LT
b. 23605-LT d. 23670-LT
____ 70. A 49-year-old presents with an abscess of the right thumb. Physician incises the abscess and purulent
sanguinous fluid is drained. The wound is packed with iodoform packing. What CPT® code(s) should be
reported?
a. 10061-F5 c. 10060-F5
b. 26010-F5 d. 26011-F5
____ 71. An 85-year-old has developed a lump in her right groin. An incision over the lesion was made and dissected
through the skin and subcutaneous tissue going deep into the femoral fascia. Sharp dissection of the mass was
performed, free from the surrounding structures. The 3-cm mass was isolated and cultured. The incision was
closed, the area was cleaned and dried, and dressing applied. What CPT® code(s) should be reported?
a. 27087 c. 27048
b. 27047 d. 27049
____ 72. A young female patient was taken to the operative suite, at which time she was placed under appropriate
anesthesia. She has been suffering from pain and potential rotator cuff tear of the right shoulder. The right arm
was sterilely draped and prepped. Arthroscopic portals were created anteriorly-posteriorly. The joint line was
carefully examined. The biceps insertion was noted to be normal. The middle and inferior glenohumeral
ligaments were visualized and noted to be normal. The undersurface of the rotator cuff was clearly visualized
and also noted to be normal. There was a large anterior spur formation. The burr was introduced through a
lateral portal and the anterior lip of the acromion was resected. The undersurface of the clavicle was noted to
be quite prominent and part of the impinging process. There was intense bursitis and a bursectomy was
performed, allowing for acromial decompression. Spurs were removed from the distal clavicle. All
instrumentation was removed, dressing was applied, and the patient was placed in a sling and returned to the
recovery room. What CPT® code(s) should be reported?
a. 23130, 23120-51 c. 23415
b. 29826, 29824-51 d. 29826
____ 73. A 14-year-old status post injury over one year ago to her left wrist presented with recurring wrist pain. The
patient was taken to the operating room and placed under general anesthesia. She was placed in wrist traction.
The radiocarpal joint was entered through sharp skin dissection and blunt dissection into the joint. There was
found to be mild synovitis in the dorsal ulnar aspect of the wrist. This was debrided arthroscopically with a
shaver. There was a peripheral tear of the triangle fibrocartilage. This area was shaved to promote healing.
Using outside-om technique, a PDS suture was placed across the TFCC and into the capsule. There was
synovitis within the midcarpal joint, but there was no articular injury. All instruments were removed and the
wounds were closed with interrupted nylon sutures. What CPT® code(s) should be reported?
a. 29844 c. 29847
b. 29845 d. 29846
____ 74. A 72-year-old female sustained a left radius fracture, resulting in volar angulation, radial shortening and loss
of radioulnar inclination. A general anesthetic was administered. A standard dorsal central approach to the
wrist was made. The capsule was opened in a T fashion and the malunion site was identified. A series of
osteotomes was utilized to open the fracture site back up and then the normal distal radial architecture was
restored. The pie-plate was then placed on the distal radius, utilizing a combination of 2.0 and 1.8 screws and
threaded pins for the distal segment and 2.7 screws proximally. Fragments were secured, and Norian SRS was
packed into the defect and allowed to harden. With this completed, the wounds were copiously irrigated with
normal saline. Soft tissue was closed back over the plate and distal radius, and secured with 2-0 Vicryl. What
CPT® code(s) should be reported?
a. 25400 c. 25607
b. 25405 d. 25609
____ 75. A patient presented with a closed, displaced supracondylar fracture of the left elbow. After conscious sedation,
the left upper extremity was draped and closed reduction was performed, achieving anatomical reduction of
the fracture. The elbow was then prepped and with the use of fluoroscopic guidance, two K-wires were
directed crossing the fracture site and purchasing the medial cortex of the left distal humerus. Stable reduction
was obtained, with full flexion and extension. K-wires were bent and cut at a 90 degree angle. Telfa padding
and splint were applied. What CPT® code(s) should be reported?
a. 24535 c. 24582
b. 24538 d. 24566
____ 76. A patient presented with a right ankle fracture. After induction of general anesthesia, the right leg was
elevated and draped in the usual manner for surgery. A longitudinal incision was made parallel and posterior
to the fibula. It was curved anteriorly to its distal end. The skin flap was developed and retracted anteriorly.
The distal fibula fracture was then reduced and held with reduction forceps. A lag screw was inserted from
anterior to posterior across the fracture. A 5-hole 1/3 tubular plate was then applied to the lateral contours of
the fibula with cortical and cancellous bone screws. Final radiographs showed restoration of the fibula. The
wound was irrigated and closed with suture and staples on the skin. Sterile dressing was applied followed by a
posterior splint. What CPT® code(s) should be reported?
a. 27814
b. 27792
c. 27766
c. 27823
____ 77. A 49-year-old female presented with chronic de Quervain’s and has been unresponsive to physical therapy,
bracing or cortisone injection. She has opted for more definitive treatment. After induction of anesthesia, the
patient’s left arm was prepared and draped in the normal sterile fashion. Local anesthetic was injected using a
combination 2% lidocaine and 0.25% Marcaine. A transverse incision was made over the central area of the
first dorsal compartment. The subcutaneous tissues were gently spread to protect the neural and venous
structures. The retractors were placed. The fascial sheath of the first dorsal compartment was then incised and
opened carefully. The underlying thumb abductor and extensor tendons were identified. The tissues were
dissected and the extensor retinaculum of the first extensor compartment was incised. The fibrotic tissue was
incised and the tendons gently released. The tendons were freely moving. Sub q tissues were closed with 3-0
Vicryl and the skin with 3-0 Prolene subcuticular closure. Steri-strips, Xeroform and dry sterile dressings
were applied. What CPT® code(s) should be reported?
a. 25001 c. 25118
b. 25000 d. 25085
____ 78. This 45-year-old male presents to the operating room with a painful mass of the right upper arm. Upon deep
dissection a large mass in the soft tissue of the patient's shoulder was noted. The mass appeared to be benign
in nature. With deep blunt dissection and electrocautery, the mass was removed and sent to pathology. What
CPT® code(s) should be reported?
a. 23076 c. 23075
b. 23066 d. 23030
____ 79. A six-year-old male suffered a fracture after falling off the monkey bars at school. He fell on an outstretched
hand and suffered a transcondylar fracture of the left humerus. After prep and drape, a manipulation was
done to achieve anatomic reduction. Once the joint was adequately reduced, skeletal traction was placed
distally and proximally to maintain excellent fixation and anatomic reduction. The pins were bent, trimmed
and covered with a sterile dressing and a posterior splint was placed on the patient’s arm. What CPT® code(s)
should be reported?
a. 24516 c. 24535
b. 24530 d. 24538
____ 80. A 27-year-old triathelete is thrown from his bike on a steep downhill ride. He suffered a severely fractured
vertebra at C5. An anterior approach is used to dissect out the bony fragments and strengthen the spine with
titanium cages and arthrodesis. The surgeon places the patient supine on the OR table and proceeds with an
anterior corpectomy at C5 with discectomy above and below. Titanium cages are placed in the resulting defect
and morselized allograft bone is placed in and around the cages. Anterior synthes plates are placed across C2-
C3 and C3-C5, and C5-C6. What CPT® code(s) should be reported?
a. 22326, 22554-51, 22845, 22851, 20930 c. 63001, 22554-51, 22845, 20931
b. 63081, 22554-51, 22846, 22851, 20930 d. 22326, 22548-51, 22846, 20931
____ 81. This 45-year-old male presents to the operating room with a painful mass of the right upper arm. General
anesthesia was induced. Soft tissue dissection was carried down through the proximal aspect of the teres
minor. Upon further dissection a large mass was noted just distal of the IGHL, which appeared to be benign in
nature. With blunt dissection and electrocautery, the 4-cm mass was removed en bloc and sent to pathology.
Wound was irrigated, repair of the teres minor with subcutaneous tissue was then closed with triple-0 Vicryl.
Skin was closed with double-0 Prolene in a subcuticular fashion. What CPT® code(s) should be reported?
a. 23076-RT c. 23075-RT
b. 23066-RT d. 11406-RT
____ 82. The patient has a torn medial meniscus. An arthroscope was placed through the anterolateral portal for the
diagnostic procedure. The patellofemoral joint showed some grade 2 chondromalacia on the patella side of the
joint only, and this was debrided with the 4.0-mm shaver. The medial compartment was also entered and a
complex posterior horn tear of the medial meniscus was noted. It was probed to define its borders. A
meniscectomy was carried out back to a stable rim. What CPT® code(s) should be reported?
a. 29880, 29879-59 c. 29870, 29879-59
b. 29870, 29877-59 d. 29881, 29877-59
____ 83. A 3-year-old is brought into the ER crying. He cannot bend his left arm after his older brother pulled it. The
physician looks at X-ray diagnosing the patient with a medial dislocated nursemaid’s elbow. The ER
physician reduces the elbow successfully. The patient is able to move his arm again. The patient is referred to
an orthopedist for follow-up care. What CPT® and ICD-9-CM codes should be reported?
a. 24640-54, 832.23, E927.0 c. 24640-54, 832.10, E927.8
b. 24565-54, 832.22, E929.8 d. 24600-54, 832.00, E928.8
____ 84. A 50-year-old male had surgery on his upper leg one day ago and is presenting with serous drainage from the
wound. He was scheduled back to the operating room for an evaluation of the hematoma. His wound was
explored and there was a hematoma at the base of the wound, which was very carefully evacuated and the
wound irrigated with antibacterial solution. What CPT® and ICD-9-CM codes should be reported?
a. 10140-79, 998.12 c. 10140-76, 998.9
b. 27603-78, 998.59 d. 27301-78, 998.12
____ 85. A 22-year-old female sustained a dislocation of the right elbow with a medial epicondyle fracture while on
vacation. The patient was put under general anesthesia and the elbow was reduced and was stable. The medial
elbow was held in the appropriate position and was reduced in acceptable position and elevated to treat non-
surgically. A long arm splint was applied. The patient is referred to an orthopedist when she returns to her
home state in a few days. What CPT® code(s) should be reported?
a. 24575-54, 24615-54-51 c. 24577-54, 24600-54-51
b. 24576-54, 24620-54-51 d. 24565-54, 24605-54-51
____ 86. A 45-year-old presents to the operating room with a right index trigger finger and left shoulder bursitis. The
left shoulder was injected with 1 cc of Xylocaine, 1 cc of Celestone, and 1 cc of Marcaine. An incision was
made over the A1 pulley in the distal transverse palmar crease, about an inch in length. This was taken
through skin and subcutaneous tissue. The Al pulley was identified and released in its entirety. The wound
was irrigated with antibiotic saline solution. The subcutaneous tissue was injected with Marcaine without
epinephrine. The skin was closed with 4-0 Ethilon suture. Clean dressing was applied. What CPT® code(s)
should be reported?
a. 26055-F6, 20610-76-LT c. 26055-F6, 20610-51-LT
b. 20552-F6, 20605-52-LT d. 20553-F6, 20610-51-LT
____ 87. A patient presents with a healed fracture of the left ankle. The patient was placed on the OR table in the
supine position. After satisfactory induction of general anesthesia, the patient’s left ankle was prepped and
draped. The tourniquet was not positioned or utilized. A small incision about 1 cm long was made in the
previous incision. The lower screws were removed. Another small incision was made just lateral about 1 cm
long. The upper screws were removed from the plate. Both wounds were thoroughly irrigated with copious
amounts of antibiotic containing saline. Skin was closed in a layered fashion and sterile dressing applied.
What CPT® code(s) should be reported?
a. 20680 c. 20670
b. 20680, 20680-59 d. 20680, 20670-59
____ 88. A patient is seen in the hospital’s outpatient surgical area with a diagnosis of a displaced comminuted fracture
of the lateral condyle, right elbow. An ORIF procedure was performed, which included the following
techniques: An incision was made in the area of the lateral epicondyle. This was carried through subcutaneous
tissue, and the fracture site was easily exposed. Inspection revealed the fragment to be rotated in two places
about 90 degrees. It was possible to manually reduce this quite easily, and the manipulation resulted in an
almost anatomic reduction. This was fixed with two pins driven across the humerus. The pins were cut off
below skin level. The wound was closed with plain catgut subcutaneously and 5-0 nylon for the skin.
Dressings and a long arm cast were applied. What CPT® and ICD-9-CM codes should be reported?
a. 24579, 29065-51, 812.52 c. 24579, 812.42
b. 24577, 812.42 d. 24575, 812.52
____ 89. A 47-year-old patient was previously treated with external fixation for a Grade III left tibial fracture. There is
now nonunion of the left proximal tibia and he is admitted for open reduction of tibia with bone grafting.
Approximately 30 grams of cancellous bone was harvested from the iliac crest. The fracture site was exposed
and the area of nonunion was osteotomized, cleaned, and repositioned. Intrafragmentary compression was
applied with three screws. The harvested bone graft was packed into the fracture site. What CPT® and ICD-9-
CM codes should be reported?
a. 27724, 733.82, 905.4 c. 27722, 733.81, 905.4
b. 27722, 733.82 d. 27724, 733.82
____ 90. A Grade I, high velocity open right femur shaft fracture was incurred when a 15-year-old female pedestrian
was hit by a car. She was taken to the operating room within four hours of her injury for thorough irrigation
and debridement, including excision of devitalized bone. The patient was then reprepped, redraped, and
repositioned. Intramedullary rodding was then carried out with proximal and distal locking screws. What
CPT® and ICD-9-CM codes should be reported?
a. 27506, 11044-51, 821.11 c. 27507, 11012, 821.01
b. 27506, 11012-51, 821.11 d. 27507, 11044, 821.10
____ 91. A 49-year-old female had two previous rotator cuff procedures and now has difficulty with shoulder function,
deltoid muscle function, and axillary nerve function. An arthrogram is scheduled to be performed. After
preparation, the shoulder was anesthetized with 1% lidocaine about 8 cc without epinephrine. The needle was
placed into the shoulder area posteriorly under image intensification. It appeared as if the dye was in the
shoulder joint. She was moved and a good return of flow was obtained. The shoulder was then mobilized and
there was no evidence of any cuff tear from the posterior arthrogram. What CPT® code(s) should be
reported?
a. 23350 c. 23350, 73040-26
b. 20610, 73040-26 d. 23350, 20610, 73040-26
____ 92. A 31-year-old secretary returns to the office with continued complaints of numbness involving three radial
digits of the upper right extremity. Upon examination, she has a positive Tinelís test of the median nerve in
the left wrist. Anti-inflammatory medication has not relieved her pain. Previous electrodiagnostic studies
show sensory mononeuropathy. She has clinical findings consistent with carpal tunnel syndrome. She has
failed physical therapy and presents for injection of the left carpal canal. The left carpal area is prepped
sterilely. A 1.5 inch 25 or 22 gauge needle is inserted radial to the palmaris longus or ulnar to the carpi radialis
tendon at an oblique angle of approximately 30 degrees. The needle is advanced a short distance about 1 or 2
cm observing for any complaints of paresthesias or pain in a median nerve distribution. The mixture of 1 cc of
1% lidocaine and 40 mg of Kenalog is injected slowly along the median nerve. The injection area is cleansed
and a bandage is applied to the site. What CPT® code(s) should be reported?
a. 20526, J3301 c. 20526, J3303
b. 20551, J3302 d. 20550, J3302
____ 93. An elderly female presented with increasing pain in her left dorsal foot. The patient was brought to the
operating room, at which time she was placed under general anesthesia. A curvilinear incision was centered
over the lesion itself. Soft tissue dissection was carried down through to the ganglion. The ganglion was
clearly identified as a gelatinous material. It was excised directly off the bone and sent to pathology. There
was noted to be a large bony spur at the level of the 1st metatarsal. Using double action rongeurs, the spur
itself was removed and sequestrectomy was performed. Following that, a rasp was utilized to smooth the bone
surface. The eburnated bony surface was then covered, utilizing bone wax. The wound was irrigated and
closed in layers. What CPT® code(s) should be reported?
a. 28122, 28090-51 c. 28045, 28090-51
b. 28111, 28092-51 d. 28100, 28092-51
____ 94. Under general anesthesia, a 45-year-old patient was sterilely prepped. The wrist joint was injected with
Marcaine and epinephrine. Three arthroscopic portals were created. The articulating surface between the
scaphoid and the lunate clearly showed disruption of the ligamentous structures. We could see soft tissue
pouching out into the joint; this was debrided. There was abnormal motion noted within the scapholunate
articulation. At this point the C-arm was brought in, arthroscopic instruments were placed in the joint and we
confirmed the location of the shaver as a probe in the scapholunate ligament. There was a significant gap
between the capitate and lunate. K-wire was utilized from the dorsal surface into the lunate, restoring the
space. Further examination revealed gross instability between the capitate and lunate. With the wrist in neutral
position, a K-wire was then passed through the scaphoid, through the capitate and into the hamate. This
provided stabilization of the wrist joint. Stitches were placed, and a thumb spica cast was applied. What
CPT® code(s) should be reported?
a. 29847 c. 29847, 29847-59
b. 29846 d. 29847, 29846
____ 95. A 74-year-old male presented with ankle avascular necrosis of the talus with collapse of the body. After
general anesthesia and sterile prep, the patient was placed prone. A lateral incision was made. The fibula was
dissected and approximately 6 cm of the fibula was removed. There were a lot of free fragments of bone
around the subtalar joint and the talus itself. Those bones were removed and there was a large defect
consistent with avascular necrosis of the body of the talus. An egg-shaped bur was introduced and the
articulating cartilage of the ankle joint was excised and debrided off. The subtalar joint was then approached
and resection of the articulating surface of the subtalar joint was completed. A bone graft was prepared on the
back table. We made two large blocks to fill the defect in the talus and then additional small fragments of
cortical cancellous bone to fill in smaller defects around the talus and ankle itself. Fixation was performed in
the calcaneocuboid. The talar screw was inserted, followed by fixation of the talonavicular, tibiotalar and
more additional compression. The ankle screws were inserted more proximally. The wound was irrigated and
closed in layers. What CPT® code(s) should be reported?
a. 28730, 20900-51 c. 28705, 20902-51
b. 28715, 20902-51 d. 28725, 20900-51
____ 96. The patient is a 17-year-old male who was struck on the elbow by another player’s stick while playing
hockey. He is found to have a fracture of the olecranon process. The patient was brought to the OR,
anesthetized and intubated. The right upper extremity was prepped with Betadine scrub and draped free in the
usual sterile orthopedic manner. The arm was then elevated and exsanguinated and the tourniquet inflated to
250 mm./Hg. A five-inch incision was made with the scalpel on the extensor side of the elbow, beginning
distally and proceeding in an oblique fashion up the proximal forearm. Dissection was carried down through
subcutaneous tissue and fascia, and bleeding was controlled with electrocautery. We then subperiostally
exposed the proximal ulna and olecranon to visualize the fracture site. The fracture could be seen at the base
of the olecranon process. We irrigated the site thoroughly and reduced the fracture fragments without
difficulty. Extending the elbow, we inserted two smooth K-wires across the fracture site. Next, through a drill
hole in the proximal ulnar shaft, we threaded an 18 gauge wire through it and wrapped it around the K-wires
in a figure-of-eight manner to further stabilize the fixation. Wires were then twisted and placed into soft
tissues. The K-wires in the olecranon were advanced slightly after being bent and cut. Sterile dressing was
applied and the patient was placed in a splint. What CPT® code(s) should be reported?
a. 24685 c. 24675
b. 24635 d. 24586
____ 97. This 56-year-old female presented with a degenerative posteromedial meniscal flap tear of the right knee.
After appropriate preoperative evaluation, the patient was taken to the operating room where general
anesthesia was instituted. The patient was placed supine on the operating table. The right lower extremity was
sterilely prepped and draped for arthroscopic surgery. The leg was exsanguinated and the tourniquet inflated.
The arthroscope was introduced first through the anterolateral portal with medial suprapatellar portal utilized.
The lateral compartment looked fairly good. There were some minimal medial degenerative changes. In the
medial compartment there was a full-thickness area of osteochondral degeneration with a flap of cartilage
noted. It was possible to remove this with a bleeding bony bed with beveled edges of cartilage. The ligament
itself was intact. The retropatellar area was normal with only Grade I chondromalacia changes noted. The
medial joint was inspected and there was a tear at the junction of the middle and posterior portions of the
meniscus, a flap tear that was based more anteriorly. This was shaved with a combination of small baskets and
punches, and the meniscal debrided back to a smooth stable rim. There was additional synovitis in the medial
aspect of the intercondylar notch and this was removed with the curved automated meniscal incisor. What
CPT® code(s) should be reported?
a. 29880, 29879-51 c. 29882
b. 29881 d. 29881, 29877-51
____ 98. This 36-year-old female presents with an avulsed anterior cruciate ligament off the femoral condyle with a
complete white on white horizontal cleavage tear of the posterior horn of the medial meniscus, causing
instability. A general endotracheal anesthesia was performed, the patient was placed supine on the operating
table and the right lower extremity was prepped with Betadine and draped free. Standard arthroscopic portals
were created and the knee was systematically examined and probed. The posterior horn of the medial
meniscus was noted to be buckled and frayed. This area was carefully probed and found to be irreparable. It
was decided that our best option was to proceed with a limited partial meniscectomy, with the goal being to
leave as much viable meniscal tissue as possible. Therefore, a medial infrapatellar portal was developed with
a longitudinal stab wound. A series of straight-angled and curved basket punches was used to perform a
saucerization of the damaged portion of the meniscus, leaving the intact portion of the medial meniscus in
place. Debris was meticulously removed with the 4.0 meniscal cutter. Approximately 50% of the medial
meniscus remained. Next, our attention was turned to the ACL repair. Through a 5 cm longitudinal anterior
incision, a central one-third tendon bone was harvested. A 10 mm graft was taken and bone plug sculpted.
Anterolateral notchplasty was done with a curet and polished with the burr. All debris was removed and
instruments were used to ensure proper isometry. The graft was tightened in extension about 2.5 mm and
actually lengthened in flexion, and this was considered acceptable. Endoscopic guides were used to create the
tibial and femoral tunnels, and the edges were rasped smooth. Using a percutaneous guide pin, the graft was
placed retrograde to the knee and secured proximally with an 8 x 25 mm interference screw. The knee was put
through range of motion, and with the leg in 30 degrees of flexion with the posterior drawer applied to the
proximal tibia, an 8 x 20 mm interference screw was used to secure the bone plug distally. The graft was tight,
isometric and without adverse features. The wound was copiously irrigated with Kantrex. Cancellous bone
fragments from bone plugs were used to graft the donor site defect in the patella. The paratenon was closed
over this to house the graft with a running #1 Vicryl. The edge of the distal bone plug was beveled with the
rongeur. The subcutaneous tissue was then closed with triple-0 Vicryl. Skin was closed with double-0 Prolene
in a subcuticular fashion. Steri-Strips, sterile dressing, cryo cuff and hinged knee brace were applied. The
patient was awakened and taken to the recovery room in satisfactory condition. What CPT® code(s) should
be reported?
a. 29888, 20902, 29880-51 c. 29888, 29881-51
b. 29888, 29882-51, 20924 d. 29889, 29880
____ 99. This patient presented with internal derangement of the left knee. After satisfactory anesthesia was
administered, the left lower extremity was prepped and draped in a sterile fashion. Routine portals were made
in the knee. We first looked at the medial compartment, which showed a complex small tear of the posterior
horn of the medial meniscus, which was debrided using a 4.0 meniscal shaver. There was an area of grade 4
chondromalacia on the proximal medial distal femur and this was all the way down to bone. There was also
evidence of chondromalacia over the patellofemoral joint of grade 4. Both areas were debrided and drilled
with a 0.45 K-wire. Multiple drill holes were placed in an attempt to get some scar tissue to form. The notch
area was normal and lateral compartment normal. Following microfracture technique, the knee was irrigated,
each portal was closed with 4-0 nylon and the patient was taken to recovery. What CPT® code(s) should be
reported?
a. 29879, 29881-59-51 c. 29879, 29879-59, 29881-51
b. 29880, 29879-51, 29877-51 d. 29879, 29877-59
____ 100. A 68-year-old female with long-standing degenerative arthritis in her right knee presented. Risks and benefits
were discussed. She was agreeable to the surgery. After adequate anesthesia was obtained, the patient was
prepped and draped in usual sterile fashion with DuraPrep and Betadine scrub. The leg was exsanguinated and
tourniquet inflated. An anterior incision was made and carried through the skin and bursa, cauterizing all
bleeders. The bursa was elevated medially and a medial parapatellar incision was made. The proximal tibia
was cleaned. The knee had an 18° flexion contracture. The cruciate ligaments were debrided along with the
menisci. The proximal tibial cutting guide was placed and the proximal tibial cut was made. The femoral
canal was entered and a 6° cut was made for the anterior jig. The distal cut was made at 6°. The femur
measured a size 2. The 2 cutting block was placed and the anterior, posterior and chamfer cuts were made.
The notch cut was made and then the trial component was placed with a size 2 tibia and 12 mm spacer and
was found to fit beautifully and it tracked well. The patella was cut and measured to be a 32. The holes were
drilled and the proximal tibial cuts were made. All the excess meniscal tissue and hypertrophic synovium
were debrided. The wound was thoroughly irrigated and the bone dried. The cement was mixed, the size 2
tibia with a 12 mm tibial tray, size 2 femur and a size 32 patella were all cemented in place removing all
excess cement. After the cement was hard, the tourniquet was released. The knee was placed through a range
of motion and was found to track beautifully. The knee was thoroughly irrigated. The retinaculum was closed
with interrupted figure-of-eight 1 Vicryl. The bursa was closed with 1 and 0. The subcutaneous layers were
closed with 0 and 2-0 and the skin with staples. Sterile dressing was applied. The patient was taken to the
recovery room in stable condition. What CPT® code(s) should be reported?
a. 27447 c. 27440
b. 27446 d. 27441
____ 101. What CPT® code(s) should be reported for open decortication and parietal pleurectomy?
a. 32220 c. 32225
b. 32320 d. 32652
____ 102. What is the largest single mass of lymphatic tissue?
a. Spleen c. Peyer’s Patches
b. Thymus d. Tonsils
____ 103. What ICD-9-CM code(s) should be reported for spontaneous pneumothorax?
a. 860.0 c. 512.8
b. 512.1 d. 860.5
____ 104. What CPT® code should be reported for a percutaneous needle biopsy of mediastinum?
a. 32662 c. 32095
b. 32405 d. 32602
____ 105. What CPT® code should be reported for a frontal sinusotomy, nonobliterative, with osteoplastic flap, brow
incision?
a. 31080 c. 31084
b. 31087 d. 31086
____ 106. What CPT® code(s) should be reported for an emergency endotracheal intubation to save the patient’s life?
a. 31502 c. 31603
b. 31500 d. 31600
____ 107. What is the name of the bony structure separating the nostrils?
a. Nasal turbinates c. Nasal membrane
b. Nasal septum d. Nasal cartilage
____ 108. What CPT® code(s) should be reported for a major thoracotomy for post-op hemorrhage?
a. 32110 c. 32310
b. 32100 d. 32120
____ 109. Johnny has a penny removed from his left nostril in the doctor’s office. What CPT® code(s) should be
reported?
a. 30320 c. 30100
b. 30300 d. 30160
____ 110. What ICD-9-CM code(s) should be reported for RSV, respiratory syncytial virus?
a. 754.0 c. 466.0
b. 471.0 d. 079.6
____ 111. What ICD-9-CM code should be reported for pyopneumothorax with fistula?
a. 510.0 c. 512.8
b. 510.9 d. 512.1
____ 112. A patient presents with wheezing and shortness of breath. After evaluating the patient, the physician
determines the patient is suffering from an exacerbation of his asthma. The physician orders nebulizer
treatments to be administered in his office. According to the ICD-9-CM guidelines for coding signs and
symptoms, what is the correct ICD-9-CM code(s)?
a. 493.92 c. 786.07, 786.05
b. 493.91, 786.07, 786.05 d. 493.91
____ 113. Provide the correct ICD 9-CM code for acute RSV bronchiolitis.
a. 466.0 c. 466.11
b. 466.19 d. 466.19, 079.6
____ 114. The provider performs a diagnostic thoracoscopy followed by the thoracoscopic excision of the pericardial
cyst. What CPT® code(s) should be reported?
a. 32601, 32662-51 c. 32658
b. 32601, 32661-51 d. 32661
____ 115. A patient’s nose was hit with a baseball during a high school baseball game. At that time reconstruction was
performed, with local grafts. Patient returns now as an adult, discontent with the bony prominence along the
bony pyramid and flat look of the tip of the nose. He underwent major repair with osteotomies and nasal tip
work. What CPT® code(s) should be reported?
a. 30410 c. 30450
b. 30435 d. 30462
____ 116. What ICD-9-CM code(s) should be reported for postoperative pulmonary edema due to fluid overload from
infusion?
a. 518.4, 276.69, E873.0 c. 518.81, 276.61, E873.1
b. 518.4, 428.1 d. 518.81, 136.3
____ 117. What ICD-9-CM code(s) should be reported for COPD with acute bronchitis?
a. 491.20 c. 491.8
b. 491.9 d. 491.22
____ 118. A patient with AML (Acute Myelogenous Leukemia) has just learned his sister is an HLA match for him.
Stem cells taken from the donor (the patient’s sister) will be transplanted into the patient to help with his
treatment. What CPT® code(s) should be used to report for the harvesting of the stem cells from the donor,
his sister?
a. 38204 c. 38206
b. 38205 d. 38207
____ 119. A patient is seen in the OR for the removal of a hepatic adenoma which has invaded the diaphragm. The
resection of the diaphragm portion of the mass was repaired with primary sutures. What CPT® code(s) should
be reported for the diaphragmatic mass resection?
a. 39540 c. 39560
b. 39545 d. 39561
____ 120. A 43-year-old female is seen in the emergency room with severe epistaxis. She said this is a common
occurrence for her during the really cold dry months of winter and this is why she is here for the third time
this week. Extensive bilateral posterior cautery and packing is required to control the hemorrhage.
What CPT® code(s) should be reported for the procedure? (Note: Do not code the E/M)
a. 30905-22 c. 30905-50
b. 30903-50 d. 30906-50
____ 121. An operative report lists excisional bilateral biopsies of deep cervical nodes and biopsy of right deep axillary
nodes as the procedures performed. The pathology report comes back confirming lymphadenitis. What CPT®
code(s) should be reported?
a. 38520-50, 38505-59, 38740-RT, 59 c. 38510-50, 38525-RT, 51
b. 38510, 38525-59 d. 38520-50, 38525-RT
____ 122. What anatomical section contains all of the thoracic viscera except the lungs?
a. Mediastinum c. Coronal plane
b. Peritoneum d. Mesentery
____ 123. A surgeon performed a transthoracic median sternotomy for exploration of the space around the lung sacs and
for drainage of fluid, caused by pneumonia. What is the appropriate code(s)for this scenario?
a. 39000, 32421-51 c. 39010
b. 39220, 32421-51 d. 39400
____ 124. An 18-month-old patient is seen in the ED unable to breath due to a toy he swallowed which had lodged in his
throat. Soon brain death would occur if an airway is not established immediately. The ED physician performs
an emergency tracheostomy, transtracheal. What CPT® and ICD-9-CM code(s) should be reported?
a. 31601, 348.89 c. 31603, 934.8
b. 31601, 31603, 938 d. 31603, 938
____ 125. A 20-year-old patient is seen for 5 transbrachial lung biopsies of 2 separate lobes. One biopsy is taken in one
lobe and 4 biopsies in another lobe. What CPT® code(s) should be reported?
a. 31628 c. 31629, 31632
b. 31628, 31632 d. 31628, 31632 x 2
____ 126. What CPT® code should be reported for an extrapleural pneumonectomy as well as empymectomy performed
during the same surgical session?
a. 32035, 32445-51 c. 32445, 32540-51
b. 32440, 32445-59 d. 32440, 32540-59
____ 127. What CPT® code should be reported for extensive excision of seven nasal polyps?
a. 30115 c. 30115 x 7
b. 30110, 30115 x 6 d. 30115, 30115-51 x 6
____ 128. A patient is seen in the endoscopy suite for a diagnostic maxillary sinusotomy. During the sinusotomy, the
physician observes some diseased tissue which needs to be removed. The physician decides to perform a
maxillary antrostomy with tissue removal. Bleeding is controlled. The patient tolerated the procedure well.
What CPT® code(s) should be reported?
a. 31231, 31267 c. 31256
b. 31254, 31256 d. 31267
____ 129. Most nasal passages have how many turbinates present on the lateral wall of each nasal cavity?
a. 2 c. 5
b. 3 d. 6
____ 130. Where in the respiratory system is the carina located?
a. Left bronchus c. Tracheal bifurcation
b. Sphenoid sinus d. Inferior turbinate
____ 131. This 25-year-old male presents with deviated nasal septum. The patient undergoes a nasal septum repair and
submucous resection. Cartilage from the bony septum was detached and the nasoseptum was realigned and
removed in a piecemeal fashion. Thereafter, 4-0 chronic was used to approximate mucous membranes. Next,
submucous resection of the turbinates was handled in the usual fashion by removing the anterior third of the
bony turbinate and lateral mucosal followed by bipolar cauterization. What CPT® code(s) should be
reported?
a. 30520, 30140-51 c. 30620, 30999-51
b. 30420, 30140-51 d. 30450, 30999-51
____ 132. This 45-year-old presents with acute pericarditis. The surgeon makes a small incision between two ribs and
enters the thoracic cavity. An endoscope is introduced and the pericardial sac is examined by direct
visualization. Using an instrument introduced through the endoscope, the surgeon creates an opening in the
pericardial sac for drainage purposes. What CPT® code(s) should be reported?
a. 32658 c. 32660
b. 32659 d. 32661
____ 133. The pulmonologist in a multispecialty group refers a patient to the otolaryngologist in the same group, same
tax ID, because they think that the shortness of breath that the patient is experiencing may be due to sinusitis
and LPR. The otolaryngologist has never seen this patient. The otolaryngologist works up the patient,
capturing a comprehensive history, detailed exam and medical decision making of moderate level. The note
indicates that he needs to perform to perform a nasal endoscopy to get a better look at what is going on in the
sinuses and a flexible laryngoscopy to determine if laryngopharyngeal (LPR) reflux is contributing to the
problems because he could not get adequate visualization on manual exam. The scopes are performed and the
otolaryngologist diagnoses chronic pansinusitis and chronic laryngitis/tracheitis and LPR. He prescribes
Singulair and Nexium and proposes endoscopic surgery be considered in the future if the current route of
treatment does not fully take care of the problems experiencing by the patient. What CPT® and ICD-9-CM
codes should be reported for the procedure? a. a. 99203-25, 31575, 473.8, 476.1
a. 99203-25, 31231, 478.8, 476.1 c. 99203-25, 31231, 478.8
b. 99203-25, 31575, 31231-51, 476.1
____ 134. A patient with recurrent pneumothoraces presents for chemopleurodesis. With the help of a thoroscope, a
catheter is inserted between the ribs and into the pleural space between the parietal and pleural viscera.
Subsequently, 5g o sterile asbestos-free talc was introduced into the pleural space via the placed catheter.
What CPT® and ICD-9-CM codes should be reported?
a. 32650, 512.1 c. 32601, 32560, 512.1
b. 32560, 512.8 d. 32650, 32560, 512.8
____ 135. A 14-year-old boy presents at the Emergency Department experiencing an uncontrolled epistaxis. Through the
nares, the ED physician packs his entire nose via anterior approach with medicated gauze. In approximately
15 minutes the nosebleed stops. What CPT® and ICD-9-CM codes should be reported?
a. 30903-50, 784.7 c. 30901, 784.7
b. 30901-50, 784.7 d. 30905, 784.7
____ 136. A 20-year-old female, who returned from spring break in Mexico six days ago, presents to the ED with a high
fever for three days, a sore throat, general aches and a miserable cough. The ED physician suspects swine flu
and orders a rapid flu test. What ICD-9-CM code(s) should be reported?
a. 488.1 c. 780.60, 462, 786.2
b. 487.1 d. 780.60, 462, 780.96, 786.2
____ 137. A 78-year-old patient with bilateral, lower lobe lung cancer has been in the hospital for seven days with a
tunneled chest tube in place to drain fluid from the pleural space. The chest tube currently is inserted between
the 4th and 5th intercostal space on the left side. There is a very bad infection at the insertion site. The
physician removes this chest tube and inserts another tube in between the 4th and 5th intrecostal space on the
right side to continue fluid drainage. The tube placed today is just like the one removed only sterile. What
CPT® and ICD-9-CM codes should be reported?
a. 32422, 162.9 c. 32552, 32551-50, 162.5. 197.0
b. 32552, 32550, 162.5 d. 32550, 162.3, 162.9
____ 138. A patient underwent bilateral nasal/sinus diagnostic endoscopy. Finding the airway obstructed the physician
fractures the middle turbinates and performs surgical endoscopy with total ethmoidectomy and bilateral nasal
septoplasty. What CPT® and ICD-9-CM codes should be reported?
a. 30930, 31255-51, 30520-51 c. 31231, 30130-51, 31255-50
b. 31255-50, 30520-50, 51 d. 31255, 30520-51
____ 139. A 55-year-old female smoker presents with cough, hemoptysis, slurred speech, and weight loss. Chest x-ray
done today demonstrates a large, unresectable right upper lobe mass, and brain scan is suspicious for
metastasis. Under fluoroscopic guidance, a needle biopsy of the lung lesion is performed for histopathology
and tumor markers. A diagnosis of small cell carcinoma is made and chemoradiotherapy is planned. What
CPT® and ICD-9-CM codes should be reported?
a. 32405, 77002-26, 786.50, 786.3, 784.5, 783.21
b. 32405, 77002-26, 162.9
c. 32405, 77002-26, 786.6
d. 32405, 77002-26, 162.3
____ 140. A surgeon performs a high thoracotomy with resection of a single lung segment on a 57-year-old heavy
smoker who had presented with a six-month history of right shoulder pain. An apical lung biopsy had
confirmed lung cancer. What CPT® and ICD-9-CM code(s) should be reported?
a. 32100, 729.5 c. 32503, 162.8
b. 32484, 162.8 d. 19271, 32551-51, 786.50
____ 141. A 3-year-old girl is playing with a marble and sticks it in her nose. Her mother is unable to dislodge the
marble so she takes her to the physician’s office. The physician removes the marble with hemostats. What
CPT® and ICD-9-CM codes should be reported?
a. 30300, 932 c. 30150, 933.0
b. 30310, 932 d. 30300, 933.0
____ 142. An ICU diabetic patient who has been in a coma for weeks as the result of a head injury, becomes conscious
and begins to improve. The physician performs a tracheostomy closure and since the scar tissue is minimal,
the plastic surgeon is not needed. What CPT® and ICD-9-CM codes should be reported for this procedure?
a. 31820, 250.30, 850.3 c. 31825, 250.30, 959.01
b. 31820, 780.01, 959.01, 250.00, V55.0 d. 31825, 250.00, 850.3, 959.01, V55.0
____ 143. A 27-year-old girl has been on the lung transplant list for months and today she will be receiving a LT and RT
lung from an individual involved in an MVA. This person was DOA at the hospital and is an organ donor. The
donor pneumonectomy was performed by physician A, the backbench work by physician B and the transplant
of both lungs into the prepped and waiting patient by physician C.
What is the correct coding for the removal (physician A), preparation (physician B) and insertion (physician
C) of the lungs?
a. 32850, 32851 x 2, 32856 c. 32850, 32851-50, 32855 x 2
b. 32850, 32851, 32855 d. 32850, 32853-50, 32856
____ 144. A patient with laryngeal spasms undergoes therapeutic injection of the vocal cords. Topical anesthesia is
administered to the oral cavity, pharynx, and larynx. Using an operating microscope, a direct laryngoscope is
inserted into the patient’s mouth. The interior larynx is examined and the surgeon injects the vocal cords at
two sites with glycerin. What CPT® and ICD-9-CM codes should be reported?
a. 31571, 478.75 c. 31571, 478.6
b. 31570, 31571, 478.74 d. 31570, 478.75
____ 145. A patient with partial vocal cord paralysis requires bilateral removal of the arytenoids cartilage to improve
breathing. The laryngoscope with operating microscope is inserted. Adequate visualization is established and
the arytenoids cartilage is exposed by excision of the mucosa overlying it. What diagnosis and procedure
code(s) should be reported for this procedure?
a. 31561, 31600, 478.33 c. 31561, 478.33
b. 31560, 478.30 d. 31560, 69990, 478.0
____ 146. A patient presents to the emergency department (ED) with a sucking chest wound. The physician on duty
performs an immediate tube thoracostomy in order to restore normal breathing to the patient before rushing
him to surgery to address other injuries. What CPT® code is used for the thoracostomy?
a. 31500
b. 32551
c. 32551, 31500-51
d. It is not coded as it will be bundled with any procedures performed during surgery.
____ 147. A patient with a diagnosis of chronic sphenoidal sinusitis undergoes a bilateral sinusotomy. While the
physician examines the diseased sphenoid sinus, she takes a biopsy of the sphenoidal masses and removes the
mucosa with several polyps. Transseptal sutures are placed and the intraoral incision is closed in a single
layer. The nose is packed and external nasal dressings are placed. What CPT® and ICD-9-CM codes should
be reported?
a. 31050-50, 30115-50, 51, 473.3 c. 31051, 12011 x 2, 473.8
b. 31050, 31051-51, 473.8 d. 31051-50, 473.3, 471.8
____ 148. A returning two-year-old child is seen in the pediatrician’s office with stridor and a “bark” like cough. The
pediatrician examines the child quickly and determines the child has croup. The child is given a neublizer
breathing treatment in the office to improve PO2 levels. Medication used is breathable epinephrine. What
CPT® and ICD-9-CM codes should be reported?
a. 94640, 478.75 c. 94642, 786.1, 478.75
b. 94640, 478.75 d. 94644, 786.1, 786.2
____ 149. The surgeon makes an incision in the neck near the cricothyroid membrane for an emergency tracheostomy
for a patient who arrives in the emergency room with tracheal crushing injuries suffered in a car accident in
which the patient was riding as the passenger. What CPT® and ICD-9-CM codes should be reported?
a. 31600, 862.39, E815.1 c. 31603, 874.02, E819.1
b. 31605, 862.29, E819.1 d. 31612, 862.39, E815.1
____ 150. A 55-year-old patient presents for a planned partial mastectomy. Two hours prior to going to the OR the
surgeon injects the patient with Technetium 99. During the mastectomy a Geiger counter probe is used to
isolate the area with the highest concentration of Technetium 99. The lymph node or nodes with the highest
concentration represent the first in the lymph node chain off the breast and are biopsied to determine if a
lymphadenectomy is necessary. As it turns out, the partial mastectomy and sentinel lymph node biopsy are the
only procedures necessary in this operative session. What CPT® code(s) should be reported?
a. 19301, 38792-51, 38525-51 c. 19302, 78195, 38792-51, 38525-51
b. 19302, 38790-51 d. 19302
____ 151. How many layers of tissue does an artery have?
a. One c. Three
b. Two d. Four
____ 152. The conduction system contains pacemaker cells, nodes, the ____ , and the
____.
a. Purkinje fibers and the Bundle of HIS. c. Heart valves and Purkinje Fibers
b. Bundle of HIS and Electrical system d. Electrical system and Bundle of HIS
____ 153. What part of the cardiovascular system is responsible for the one way flow of blood through the chambers of
the heart?
a. Septum c. Bundle of HIS
b. Heart valves d. Atria
____ 154. Which main coronary artery bifurcates into two smaller ones?
a. Right c. Inverted
b. Left d. Superficial
____ 155. What is the term for the divider between the heart chamber walls?
a. SA node c. Septum
b. Bundle branch d. Mitral
____ 156. A patient suffering from an abdominal aortic aneurysm involving a renal artery undergoes endovascular repair
using modular prosthesis with two docking limbs. Select the CPT® cod(s) for this procedure.
a. 34805 c. 34803
b. 0078T, 0079T d. 34802
____ 157. A physician places a centrally inserted, tunneled central venous access device with a subcutaneous pump on a
7 year-old patient.
a. 36561 c. 36560
b. 36563 d. 36558
____ 158. Patient presents to her physician 10 weeks following a true posterior wall myocardial infarction. The patient is
still symptomatic. What is the correct ICD-9-CM code for this condition?
a. 414.8 c. 410.60
b. 410.62 d. 412
____ 159. ____ is a term that stands for enlargement of the heart.
a. Cardiorenal c. Cardiomegaly
b. Angiomegaly d. Valvuloplasty
____ 160. Repair of coronary vessel is called:
a. Endarterectomy c. Aortic
b. Angioplasty d. Endovascular
____ 161. A physician performs a four-vessel autogenous (one venous, three arterial) coronary bypass on a patient who
had a previous CABG two years ago, utilizing the saphenous vein and the left and right internal mammary
arteries. Select the CPT® codes for this procedure.
a. 33535, 33510-51, 33530, 35600 c. 33535, 33517, 33530, 35600
b. 33534, 33518, 33530 d. 33535, 33517, 33530
____ 162. A patient presented to the ED and was found to have a ruptured abdominal aortic aneurysm. He was taken to
emergency surgery where a physician performed a direct repair. The physician documented that the aneurysm
involved the common iliac. Select the proper CPT® code for this procedure.
a. 34800 c. 35103
b. 35092 d. 35102
____ 163. A patient presents to the hospital for a cardiovascular SPECT study. A single study is performed under stress,
but without quantification, with a wall motion study, and ejection fraction. Select the CPT® code(s) for this
procedure.
a. 78451, 78472 c. 78453
b. 78453, 78472 d. 78451
____ 164. Intracoronary stents are placed in the right coronary and left anterior descending arteries for a patient with
stenosis. Percutaneous transluminal balloon angioplasty is performed on the left circumflex coronary artery.
Choose the correct CPT® codes for this procedure.
a. 92980-RC, 92981-LD, 92984-LC c. 92980, 92981
b. 92980 X 2, 92982 d. 92980-RC, 92981-LD, 92982-LC
____ 165. Select the ICD-9-CM diagnosis codes used for pseudoaneurysm, cardiac tamponade and left ventricular
failure?
a. 442.9, 423.3, 428.1 c. 424.0, 420.99, 428.1
b. 414.10, 420.99, 428.0 d. 441.9, 423.9, 428.0
____ 166. Physician changes the battery on a patient’s dual chamber permanent pacemaker.
a. 33212 c. 33213-52, 33233-51
b. 33213-52 d. 33213, 33233-51
____ 167. Physician replaces a single chamber permanent pacemaker with a dual chamber permanent pacemaker.
a. 33213, 33233-51 c. 33214
b. 33213, 33233-51, 33235-51 d. 33212, 33233-51
____ 168. Patient presents to his physician’s office. He is diagnosed with benign hypertension and Stage 3 chronic
kidney disease.
a. 403.10, 585.3 c. 403.10, 585.6
b. 401.1, 585.3 d. 401.1, 585.9
____ 169. In the cath lab, a physician places a catheter in the aortic arch from a right femoral artery puncture.
Fluoroscopic imaging performed by the physician.
a. 36215, 75605-26 c. 36200, 75650-26
b. 36200, 75605-26 d. 36215, 75650-26
____ 170. In the cath lab the following procedures are performed: Catheter placed in the left renal, accessory renal
superior to the left renal and one main right renal artery. Radiologic supervision and imaging s performed in
all locations.
a. 36245, 36245-59, 75724-26
b. 36245, 36424-59, 364245, 59
c. 36245, 36245-59, 36245-59, 75724-26, 72722-26-LT
d. 36245-LT, 36245-59-LT, 36245-59-RT, 75724-26, 75774-26
____ 171. Patient is diagnosed with acute systolic heart failure due to hypertension with CKD stage IV.
a. 404.91, 428.21, 585.4 c. 401.9, 403.90, 428.21, 585.4
b. 402.91, 403.90, 428.21, 585.4 d. 403.90, 428.21, 585.4
____ 172. A patient presents for epicardial placement of a permanent pacemaker via median sternotomy to the right
atrium and ventricle. The patient has bundle branch block and nodal dysfunction.
a. 33214, 426.51, 427.89 c. 33202, 33213-51, 426.50, 427.81
b. 33203, 33213-51, 426.53, 427.81 d. 33208, 33213-51, 426.50, 427.81
____ 173. A physician states he performed a comprehensive EP study with induction of arrhythmia in the hospital. The
report shows bundle of HIS recording, pacing and recording of the right atrium, and induction of arrhythmia
by electrical pacing.
a. 93600-26, 93602-26, 93610-26, 93618-26 c. 93620-26, 93621-26
b. 93620-26 d. 93619-26
____ 174. Due to infections from hemodialysis, the physician replaces a dual chamber cardioverter-defibrillator system
with an epicardial dual chamber cardioverter defibrillator system.
a. 33249 c. 332214
b. 33249, 33241-51, 33243-51, 33244-51 d. 33244, 33202-51, 33240-51, 33241-51
____ 175. A physician supervises a patient during a cardiac stress test performed at the hospital and writes the
interpretation and report.
a. 93015 c. 93016, 93018
b. 93016-26 d. 93016, 93017
____ 176. Aortography and bilateral extremity angiography was performed. The physician placed the catheter in aorta
at level of the renal arteries and injected contrast for the aortography and repositioned the catheter just above
the bifurcation for angiography of the extremities.
a. 36200, 75630-26 c. 36200, 75625-26, 75710-50-26
b. 36200, 75625-26, 75716-26 d. 36200, 75716-26
____ 177. Catheter advanced from the right femoral artery into the left and right pulmonary artery. The catheter was
further negotiated into the right lower lobe. Pulmonary angiography performed in all locations.
a. 36014, 36013-59, 75743
b. 36015, 36013-59, 75743, 75774
c. 36015-RT, 36014-59-LT, 75743-26, 75774-26
d. 36015, 36014-59, 75743
____ 178. An arterial catheterization is performed by cutdown for transfusion.
a. 36600 c. 36625
b. 36620 d. 36640
____ 179. A PICC with a port is placed for a 45 year-old patient for chemotherapy infusion under fluoroscopic guidance
in the hospital.
a. 36568, 77001 c. 36570, 77001-26
b. 36571, 77001-26 d. 36571, 77001
____ 180. A physician inserts a centrally inserted, tunneled central venous access device with a subcutaneous pump on a
7 year-old.
a. 36561 c. 36560
b. 36563 d. 36558
____ 181. The cardiologist advances a 6 French catheter into the left renal artery via a right common femoral puncture.
It is selectively catheterized and angiographic films are taken. The catheter was then removed and a
diagnostic guiding type, RDC catheter was used and the left renal artery was selectively engaged. A 0.014
Supracore wire was used and the lesion was crossed. A 6.0 X 18 mm balloon expandable Racer stent was
introduced. This was expanded around 8 atmospheres of pressure which is nominal. Angiography revealed
excellent results with no residual stenosis.
a. 36217, 36218, 37205, 37206, 75710-26, 75665-26, 75960-26
b. 36245-LT, 37205, 75960-26
c. 36245, 36245-59, 75710-26, 75658-26, 75960-26, 75962-26
d. 36217-RT, 36217-LT, 37205, 75724-26, 75960-26
____ 182. Preoperative Diagnosis: Aortic valve stenosis with coronary artery disease associated with congestive heart
failure
Postoperative Diagnosis: Same
Anesthesia: General endotracheal
Incision: Median sternotomy
Description of Procedure: The patient was brought to the operating room and placed in supine position. After
the patient was prepared, median sternotomy incision was carried out and conduits were taken from
the left arm as well as the right thigh. She was cannulated after the aorta and atrium were exposed and full
heparinization.

She went on cardiopulmonary bypass and the aortic cross-clamp was applied Cardioplegia was delivered
through the coronary sinuses in a retrograde manner. The patient was cooled to 32 degrees. Iced slush was
applied to the heart. The aortic valve was then exposed through the aortic root by transverse incision. The
valve leaflets were removed and the #23 St. Jude mechanical valve was secured into position by
circumferential pledgeted sutures. At this point, aortotomy was closed.

Attention was turned to the coronary arteries. The first obtuse marginal artery was a very large target and the
vein graft to this target indeed produced an excellent amount of flow. Proximal anastomosis was then carried
out to the foot of the aorta. The radial artery was anastomosed to the left anterior descending artery target in
an end-to-side manner. The proximal anastomosis was then carried out to the root of the aorta.

The patient came off cardiopulmonary bypass after aortic cross-clamp was released. She was adequately
warmed. Protamine was given without adverse effect. Sternal closure was then done using wires. The
subcutaneous layers were closed using Vicryl suture. The skin was approximated using staples.
a. 33400, 33533, 33510 c. 33405, 33533, 33510, 35500
b. 33405, 33533, 33517, 35600 d. 33411, 33533, 33517, 35600
____ 183. During an inpatient hospitalization, a patient who suffered a myocardial infarction had a combined right and
left heart catheterization. Access was achieved through the right femoral artery and the right femoral vein.
Selective catheterization of the coronary arteries and selective catheterization of the left ventricle were
followed by injections of contrast and angiography. During the right heart catheterization, angiography of the
right atrium was performed. Imaging supervision, interpretation and report for all angiography was done
during the cardiac catheterization. Select the CPT® codes for this procedure.
a. 93453-26, c. 93460
b. 93460-26, 93565-26 d. 93460, 93565
____ 184. A 35 year-old patient presented to the ASC for PTA of an obstructed hemodialysis AV graft in the venous
anastomosis and the immediate venous outflow. The procedure was performed under moderate sedation
administered by the physician that performed the PTA. The physician performed all aspects of the procedure,
including radiological supervision and interpretation. Code for all services performed.
a. 35460, 99144, 75978-26 c. 35476, 75978-26
b. 35492, 75978-26 d. 35476, 99144, 75978-26
____ 185. What is included in all vascular injection procedures?
a. Catheters, drugs, and contrast material
b. Selective catheterization
c. Just the procedure itself
d. Necessary local anesthesia, introduction of needles or catheters, injection of contrast
media with or without automatic power injection, and/or necessary pre-and postinjection
care specifically related to the injection procedure.
____ 186. Patient undergoes transcatheter placement of an extracranial vertebral artery stent in the right vertebral artery.
a. 0075T c. 35005
b. 35301 d. 0075T-26
____ 187. Catheter advanced from the right femoral artery into the left and right pulmonary artery. The catheter was
further negotiated into the right lower lobe. Pulmonary angiography performed in all locations, including
radiologic supervision and interpretation.
a. 36015-RT, 36014-59-LT, 75743-26, 75774-26
b. 36015-50, 36014, 75743-26
c. 36014-50, 75741, 75774-26
d. 36015, 36014-59, 75741-26, 75741-59
____ 188. INDICATIONS FOR CORONARY INTERVENTION: Acute inferior myocardial infarction. Documented
mildly occlusive plaque with much clot in the right coronary artery.
PROCEDURE: Insertion of temporary pacemaker in the right femoral vein. Primary stenting of the right
coronary artery with a 4.5 x 16 mm Express stent. Angio-Seal to the vessels of the right common femoral
artery post procedure, and also Angio-Seal of the right common femoral vein.
TECHNIQUE: Judkins percutaneous approach from the right groin with Perclose at the arterial puncture site
post procedure.
CATHETERS: #4-French Angio-Jet catheter device, insertion of a #5-French temporary pacing wire, a 4.5 x
16 mm Express stent.
PRESSURES: Aortic Pressure: 107/78

RESULTS:
Coronary stenting procedure of the right coronary artery: The right coronary artery was primarily stented
with a 4.5 x 16 mm Express stent. It was expanded to 12 atmospheres. There was no residual stenosis.

IMPRESSION: Successful Angio-Jet and stenting of the distal right coronary artery with no residual stenosis.
Angio-Seal to the right femoral vein post procedure.
PROCEDURE: Through the femoral artery sheath the EBU was advanced to the left main. Following this, a
PT graphic intermediate wire was used to cross the lesion. Following this, angioplasty of the lesion was
performed utilizing a 2.5 x 20 millimeter CrossSail balloon at multiple sites to ten atmospheres. Following
this, there was a fair result, however there was a significant stenosis and significant calcification at the area
and the decision was made to pursue trying to stent the lesion. Multiple stents were attempted including a 2.5
x 9 millimeter zipper MX and a 2.5 x 13 millimeter Guidant stent. This was abandoned and in switching out
to a balloon for further ballooning, the patient became hypertensive and with difficulty in terms of her
respiratory status. Angiography revealed an occlusion of the mid left anterior descending and thrombus
throughout the proximal left anterior descending extending into the left main. Recheck of ACT showed the
ACT to be at eight seconds. This likely represented subtherapeutic range for her anticoagulation. A check of
her medications revealed that instead of Angiomax, the patient had been given ReoPro without antithrombotic
agent. She was therefore given IV heparin up to 12,000 units and her ReoPro was continued. The lesion was
then rewired and an AngioJet was used to try to suction out this area of thrombus.
Unfortunately the AngioJet was unable to cross the mid left anterior descending lesion and therefore was
somewhat limited in its use for a more distal thrombus, it did suction out the proximal left anterior descending
thrombus. At this point, the patient was emergently intubated and multiple pressors were started including
dopamine, Levophed, vasopressin and epinephrine. Following this, a laser was attempted to cross the lesion
an excimer laser X80 Spectranetics 0.9 Vitesse, however, this laser was unable to cross the lesion. Therefore
a long balloon, a 2.0 x 40 millimeter CrossSail balloon was used to cross the lesion and inflate multiple
segments of the mid left anterior descending up to a maximum inflation pressure of ten atmospheres. This
improved flow, though by no means restored it back to normal. Therefore following this, longer balloon
inflations were performed utilizing a 2.0 x 20 millimeter CrossSail balloon up to fourteen atmospheres for one
and a half minutes. This did not improve significantly the flow distally and therefore the decision was made
to try to stent the mid segment with a 2.5 x 9 millimeter zipper MX stent to a maximum inflation pressure of
fourteen atmospheres. This resolved the issue in terms of the mid left anterior descending lesion, however
beyond the stent there was continued to be residual stenosis and multiple balloons were used to balloon this
up to a 2.5 x 20 millimeter balloon up to fourteen atmospheres. The final result in the left anterior descending
revealed a lesion in the mid left anterior descending that was approximately 40 percent, there was TIMI III
flow throughout the proximal and mid left anterior descending. However, at the level of the apex, there was
TIMI 0 flow. Throughout the angioplasty, the patient had episodes of bradycardia and a temporary pacemaker
was placed, and this was removed at the end of the procedure.

IMPRESSION: Successful stent to the mid left anterior descending, complicated by thrombotic event in the
left anterior descending system. Final result was a successful stent to the mid left anterior descending with
residual TIMI 0 flow in the distal left anterior descending. At the end of the case an intra-aortic balloon was
placed in the left femoral artery sheath and the patient was sent to the Coronary Care Unit on multiple
pressors including epinephrine, vasopressin, Levophed and dopamine.
a. 92980-RC, 92981-RC c. 92980-LC, 92981-RC, 92973
b. 92980-RC, 92981-LD, 33967, 92973 d. 92982-RC, 92980-RC, 92973-RC
____ 189. A patient has a complete TTE performed to assess her mitral valve prolapse (congenital). The physician
performs the study in his cardiac clinic.
a. 93303 c. 9308
b. 93306 d. 93312
____ 190. A patient has a Transtelephonic rhythm strip pacemaker evaluation for his dual chamber pacemaker started.
The physician plans on it being worn for 90 days. After 2 months, the patient no longer wants to wear the
device and it is removed. What can the physician report?
a. 93293-52 c. 93296
b. 93295 d. 93293
____ 191. A patient is brought to the operating suite when she experiences a large output of blood in her chest tubes post
CABG. The physician that performed the original CABG yesterday is concerned at the post-operative
bleeding. He explores the chest and finds a leaking anastomosis site that he resutured.
a. 35761 c. 35820-78
b. 35761-78 d. 35241
____ 192. MAZE procedure is performed on a patient with atrial fibrillation. The physician isolates and ablates the
electric paths of the pulmonary veins in the left atrium, the right atrium, and the atrioventricular annulus while
on cardiopulmonary bypass.
a. 33254 c. 33256
b. 33255 d. 33259
____ 193. Patient undergoes a mitral valve repair with a ring insertion and an aortic valve replacement, on
cardiopulmonary bypass.
a. 33464, 33406-51 c. 33430, 33405-51
b. 33426, 33405-51 d. 33468, 33426-51
____ 194. Patient undergoes a 3 artery CABG. A surgical assistant procures the artery used for the grafts. Code for the
assistant surgeon.
a. 33535-80 c. 33510-80
b. 33533-80, 35600-80 d. 33517-80, 35600-80
____ 195. The skin over the left groin was prepped and draped in a sterile fashion and anesthetized with 1% Xylocaine.
A 5 French pigtail catheter was placed in the abdominal aorta and a run-off was performed following injection
of 80cc of contrast. Oblique DSA images of the iliac circulation were performed following 2 injections, each
15cc.

Findings: Abdominal aorta: no signs of renal artery stenosis. There is mild atheromatous change involving
the lower abdominal aorta. There are 2 eccentric plaques arising from the distal aorta just above the iliac
bifurcation. There are high-grade stenoses involving both proximal iliacs, the right far more pronounced than
the left.

The right superficial femoral, profunda femoral, popliteal arteries are normal. The trifurcation vessels are
unremarkable.

On the left, there is an eccentric plaque in the common femoral artery just below the catheter entrance site.
This creates approximately 40-50% stenosis at this site. The remainder of the proximal femoral artery is
normal. The trifurcation vessels and popliteal artery are normal.
a. 36200, 75650, 75625 c. 36200, 75630-26
b. 36215, 75630-26-50 d. 36200, 75625, 75600
____ 196. PREOPERATIVE DIAGNOSIS: Heart Block
POSTOPERATIVE DIAGNOSIS: Heart Block
ANESTHESIA: Local anesthesia
NAME OF PROCEDURE: Reimplantation of dual chamber pacemaker
DESCRIPTION: The chest was prepped with Betadine and draped in the usual sterile fashion. Local
anesthesia was obtained by infiltration of 1% Xylocaine. Then a subfascial incision was made about 2.5 cm
below the clavicle and the old pulse generator was removed. Using the Seldinger technique, the subclavian
vein was cannulated and through this, the old atrial lead was removed and a new atrial lead (serial #
6662458) was placed in the left atrium, and to the atrial septum. Thresholds were obtained as follows: the P-
wave was 1.4 millivolts, atrial threshold was 1.6 millivolts with a resultant current of 3.5 mA, and resistance
of 467 ohms.
Using a second subclavian stick in the Seldinger technique, the old ventricular lead was removed and a new
ventricular lead (serial # 52236984) was inserted and placed into the right ventricular apex. The thresholds
were obtained and were as follows: R-wave was 23.5 millivolts. The patient was pacing at 100% at 0.5 volts
with resultant current of 0.8 mA and resistance of 480 ohms. When we were satisfied with the thresholds, the
leads were connected to the pacemaker generator (serial # 22561587) which was inserted into the previously
created pocket.
The wound was thoroughly irrigated with antibiotic solution and hemostasis was obtained. The incision was
closed in layered fashion with 2-0 Dexon. A compressive dressing was applied and the patient tolerated the
procedure very well. He was taken to the recovery room in satisfactory condition.
a. 33207, 33206-51, 33236-51 c. 33208, 33238-51, 33241-51
b. 33202, 33233-51 d. 33235, 33208-51, 33233-51
____ 197. A patient presents for extremity venous study. Complete noninvasive physiologic studies of both lower
extremities.
a. 92922 c. 93965
b. 93923 d. 93965-50
____ 198. An electrophysiologist performs the following EP study in the cardiac suite: right atrial and ventricular
pacing, recording of the bundle of HIS, right atrial and ventricular recording, and left atrial and ventricular
pacing and recording from the left atrium.
a. 93600, 93602, 93603, 93610, 93612, 93618, 93621, 93622
b. 93619, 93621
c. 93620, 93621, 93622
d. 93620, 93618, 93621
____ 199. CLINICAL SUMMARY: The patient is a 41-year-old female with known coronary disease and recent
recurrent chest pain, cardiac catheterization demonstrating subtotal occlusion of the diagonal artery at its
takeoff from the left anterior descending artery with the diagonal artery taking off within the left anterior
descending vessel.

PROCEDURE: With informed consent obtained, the patient was prepped and draped in the usual sterile
fashion. With the right groin area infiltrated with 2% Xylocaine and the patient given 2mg of Versed and
50mcg Fentanyl intravenously for conscious sedation and pain control. The right femoral artery was
cannulated with a modified Seldinger technique and a 6-French catheter sheath placed. A 6-French JL3.5
catheter with no side holes was utilized as a guiding catheter. After the initial guiding picture had been
obtained, the patient was given Angiomax per protocol, and a short Cross-it 100 wire was advanced to the
LAD and then into the diagonal vessel. A 2.0. 15-mm-long Maverick balloon was used for dilatation of the
diagonal artery ostium with inflation pressure up to 8 atmospheres was applied. Final angiographic
documentation was carried out after the patient received 200 mcg of intracoronary nitroglycerine. The guiding
catheter was then pulled, the sheath secured in place. The patient is now being transferred to telemetry for
post coronary intervention observation and care.

RESULTS: The initial guiding picture of the left coronary system demonstrates the high-grade ostial stenosis
of the diagonal artery taking off within the LAD. Following the coronary intervention with balloon
angioplasty there is complete resolution of the stenosis with less than 10% residual narrowing observed, no
evidence for intimal disruption, no intraluminal filling defect, and good antegrade TIMI III flow preserved.

CONCLUSION: Successful coronary intervention with balloon angioplasty to the ostial/proximal segment of
the second diagonal vessel.
a. 92980-LD c. 92980-LD, 92984-LD
b. 92982-LD d. 92995
____ 200. Preoperative Diagnosis: Coronary artery disease associated with congestive heart failure; in addition, the
patient has diabetes and massive obesity.
Postoperative Diagnosis: Same
Anesthesia: General endotracheal
Incision: Median sternotomy

Indications: The patient had presented with severe congestive heart failure associated with her severe
diabetes. She had significant coronary artery disease consisting of a chronically occluded right coronary
artery but a very important large obtuse marginal artery coming off as the main circumflex system.

She also has a left anterior descending artery which has moderate disease and this supplies quite a bit of
collateral to her right system. The decision was therefore made to perform a coronary artery bypass grafting
procedure, particularly since she is so symptomatic. The patient was brought to the operating room

Description of Procedure: The patient was brought to the operating room and placed in supine position.
Myself, the operating surgeon was scrubbed throughout the entire operation. After the patient was prepared,
median sternotomy incision was carried out and conduits were taken from the left arm as well as the right
thigh. The patient weighs almost three hundred pounds and with her obesity there was some concern as to
taking down the left internal mammary artery. Because the radial artery appeared to be a good conduit she
would have arterial graft to the left anterior descending artery territory. She was cannulated after the aorta and
atrium were exposed and full heparinization.

Attention was turned to the coronary arteries. The first obtuse marginal artery was a very large target and the
vein graft to this target indeed produced an excellent amount of flow. Proximal anastomosis was then carried
out to the foot of the aorta. The left anterior descending artery does not have severe disease but is also a very
good target and the radial artery was anastomosed to this target and the proximal anastomosis was then
carried out to the root of the aorta.

Sternal closure was then done using wires. The subcutaneous layers were closed using Vicryl suture. The skin
was approximated using staples.
a. 33533, 33510 c. 33533, 33517
b. 33511 d. 33533, 33517, 35600
____ 201. CLINICAL SUMMARY: The patient is a 55-year-old female with known coronary disease and previous left
anterior descending and diagonal artery intervention, with recent recurrent chest pain. Cardiac catheterization
demonstrated continued patency of the stented segment but diffuse borderline changes in the ostial/proximal
portion of the right coronary artery.

PROCEDURE: With informed consent obtained, the patient was prepped and draped in the usual sterile
fashion. With the right groin area infiltrated with 2% Xylocaine and the patient given 2 mg of Versed and 50
mcg of fentanyl intravenously for conscious sedation and pain control, the 6-French catheter sheath from the
diagnostic study was exchanged for a 6-French sheath and a 6- French JR4 catheter with side holes utilized.
The patient initially received 3000 units of IV heparin, and then IVUS interrogation was carried out using an
Atlantis Boston Scientific probe. After it had been determined that there was significant stenosis in the
ostial/proximal segment of the right coronary artery, the patient received an additional 3000 units of IV
heparin as well as Integrilin per double-bolus injection. A 3.0, 16-mm-long Taxus stent was then deployed in
the ostium and proximal segment of the right coronary artery in a primary stenting procedure with inflation
pressure up to 12 atmospheres applied. Final angiographic documentation was carried out and then the
guiding catheter pulled, the sheath upgraded to a 7-French system because of some diffuse oozing around the
6-French-sized sheath, and the patient is now being transferred to telemetry for post coronary intervention
observation and care.
RESULTS: The initial guiding picture of the right coronary artery demonstrates the right coronary artery to be
dominant in distribution, with luminal irregularities in its proximal and mid third with up to 50% stenosis in
the ostial/proximal segment per angiographic criteria although some additional increased radiolucency
observed in that segment.

IVUS interrogation confirms severe, concentric plaque formation in this ostial/proximal portion of the right
coronary artery with over 80% area stenosis demonstrated. The mid, distal lesions are not significant with less
than 40% stenosis per IVUS evaluation.
Following the coronary intervention with stent placement, there is marked increase in the ostial/proximal right
coronary artery size with no evidence for intimal disruption, no intraluminal filling defect, and TIMI III flow
preserved.

CONCLUSION: Successful coronary intervention with drug-eluting Taxus stent placement to the
ostial/proximal right coronary artery.
a. 92980-RC, 92978-RC c. 92980-RC, 92978-51-RC
b. 92980-RC, 92984-RC, 92978-RC d. 92982-RC, 92981-RC, 92978-51-RC
2011 Surgery Section Part II
Answer Section

MULTIPLE CHOICE

1. ANS: C
Rationale: Urticaria can also be described as hives and shows on the skin as raised, red, itchy wheals.

PTS: 1 DIF: Easy


2. ANS: D
Rationale: Scattered throughout the basal layer of the epidermis are cells called melanocytes, which produce
the pigment melanin, one of the main contributors to skin color.

PTS: 1 DIF: Easy


3. ANS: C
Rationale: The hypodermis is also known as subcutaneous fascia and is made up of fat and connective tissue
responsible for binding skin to the tissue beneath it.

PTS: 1 DIF: Easy


4. ANS: A
Rationale: A keloid scar is excess growth of the connective tissue during the healing process.

PTS: 1 DIF: Easy


5. ANS: B
Rationale: Carbuncles and furuncles (boils) typically are caused by a staphylococcal infection. Several
furuncles together make up a carbuncle and often involve a group of hair follicles.

PTS: 1 DIF: Easy


6. ANS: A
.
Rationale: Two layers make up human skin: the dermis and the epidermis. Some textbooks refer to the
hypodermis as a layer of the skin. The hypodermis is the tissue that connects the skin to the underlying tissue,
which is technically, not part of the skin.

PTS: 1 DIF: Easy


7. ANS: C
Rationale: Melanoma in-situ is not found in the neoplasm table, it is necessary to look up melanoma, then
arm in the ICD-9-CM alphabetic index.

PTS: 1 DIF: Easy


8. ANS: B
Rationale: Use the Neoplasm Table in the ICD-9-CM alphabetic index. Look under skin. Since this is a
metastasis of a prior CA, use the code from the secondary column.

PTS: 1 DIF: Easy


9. ANS: B
Rationale: In the ICD-9-CM Index, look for Complications, surgical procedures, wound infection. Although
there is no mention of an infection, under code 998.59, there is a nonessential modifier stating postoperative
wound.
PTS: 1 DIF: Easy
10. ANS: D
Rationale: The pathology report indicates the lesion is dysplastic, which would be classified in the neoplasm
table under Uncertain Behavior.

PTS: 1 DIF: Easy


11. ANS: B
Rationale: In the CPT® Index, look for Removal, Nail and you are directed to two code ranges.
Documentation states that the entire nail and root (nail matrix) are removed. Removal of the nail matrix is
what sets this code apart from 11730 where only the nail is removed. There is no mention of
excising/amputation of any bone (tuft of distal phalanx), which is included in 11752.

PTS: 1 DIF: Moderate


12. ANS: D
Rationale: In the Index, look for Destruction, Warts and you are directed to CPT® code range 17110-17111.
The guidelines under Destruction state flat wards and plantar warts are both included in the definition of
lesions. Warts are considered benign lesions so they are code from code range 17110-17111. A total of 15
lesions were destroyed by cryosurgery. Code 17111 represents the destruction of 15 or more lesions.

PTS: 1 DIF: Moderate


13. ANS: D
Rationale: Cryosurgery is a method of destruction where extreme cold is used to destroy the lesion. They are
selected based on the type of lesion and size or number. A molluscum contagiosum is a benign lesion. Based
on the number of lesions (16) removed, 17111 is the correct code.

PTS: 1 DIF: Moderate


14. ANS: A
Rationale: Look in CPT® under Pedicle Flap, formation, and you are directed to 15570-15576. Code
selection is based on location. Subsection guidelines for Flaps state that the codes refer to the recipient site
not the donor site. The term “pedicle” indicates that this is a flap not a direct graft, where skin is removed
from one site and transferred to another. Instead, a “flap” of skin is raised, leaving it attached to its source
location to maintain blood supply until it is established sufficiently in the new site. Code 15574 describes a
direct pedicle graft of the hands with or without transfer.

PTS: 1 DIF: Moderate


15. ANS: D
Rationale: ICD-9-CM Guidelines 1.C.17.c.1 state that when more than one burn is present to sequence first
the code that reflects the highest degree of burn. Third degree burns to the left leg below the knee extending
to the foot (multiple sites) would be coded as 945.39; third degree burns to the left side of the chest would be
coded as 942.32; second degree burns to the posterior back would be coded 942.24; and second degree burns
to the left arm would be coded 943.20. Category 948 is used to identify the extent of the body surface
involved. The fourth digit identifies the total body surface area (TBSA) involved (all degrees); the fifth digit
identifies the percentage of body surface involved in third-degree burns. The TBSA is 22.5% and third degree
is 9%. This would be coded with 948.20.

PTS: 1 DIF: Moderate


16. ANS: C
Rationale: The patient is presenting with acne, additionally the patient has a suspicious lesion the physician
has taken a biopsy of. Since this is being submitted to the carrier prior to the pathology report it is necessary
to report Unspecified for the lesion.

PTS: 1 DIF: Moderate


17. ANS: B
Rationale: Using the index in CPT® under injection, lesion you are directed to CPT 11900-11901. Code
selection is based on the number of lesions treated, not the number of injections. In this case one lesion is
treated, making 11900 the correct code. Using the HCPCS Index or the Table of Drugs you will find the code
J3301 for the Kenalog 40-mg. Verify the code in the J code section. Using the ICD-9-CM alphabetic index
you’ll find that Keloid directs you to 701.4, Keloid scar. Verify the code in the Tabular List.

PTS: 1 DIF: Moderate


18. ANS: B
Rationale: In the CPT® Index, look for Shaving, Skin Lesion and you are directed to range 11300-11313.
Code selection is based on location and size. This lesion is on the left cheek narrowing the range to 11310-
11313. The size is 1.8 cm making 11312 the correct code choice.

PTS: 1 DIF: Moderate


19. ANS: C
Rationale: This is excision of a benign lesion of the face. In the CPT® Index, look for Excision, Skin,
Lesion, Benign and you are directed to code range 11400-11471. Code selection is based on location and
size. The lesion is on the face, narrowing the code selection to 11440-11446. The lesion size is documented
as .8 cm plus .2 cm for the margins for the complete excision to be 1.0 cm of the face making 11441 the
correct code. The guidelines for excisions state if the repair is intermediate (layers), a code from range
12031-12057 should be reported in addition to the excision code. Code selection is again based on location
and size. 12051 is the correct repair code. Modifier 51 is used to indicate multiple procedures. For the
diagnosis, use the Neoplasm table and look under skin, jaw, benign which directs you to 216.3.

PTS: 1 DIF: Moderate


20. ANS: B
Rationale: Seborrheic keratosis is a benign lesion. In the CPT® Index look for Excision, Skin, Lesion,
Benign and you are directed to code range 11400-11471. Code selection is based on location and size. The
right axilla is on the trunk underneath the arm narrowing our code selection to 11400-11406. 3 mm converts
to 0.3 cm making the code selection 11400. The closure is a simple closure so it is included in the excision.
In the ICD-9-CM look for Keratosis, seborrheic and you are directed to 702.19.

PTS: 1 DIF: Moderate


21. ANS: D
Rationale: In the CPT® Index, look for Removal, Skin Tags and you are directed to code range 11200-11201.
Code selection is based on the number of skin tags removed. A total of 59 skin tags were removed. Code
11200 is reported for the first 15. 11201 is reported for each remaining 10 (or part thereof). 11200, 11201 x 5
is correct. Modifier 51 is not required for an add-on code. Since the code description of 11201 includes “part
thereof” it is not necessary to append a modifier 52 if the full 10 is not reached. In the ICD-9-CM look for
Tag, skin and you are directed to 701.9.

PTS: 1 DIF: Moderate


22. ANS: B
Rationale:: Squamous Cell Carcinoma is a malignant neoplasm. In the CPT® Index, look for Excision, Skin,
Lesion, Malignant and you are directed to code range 11600-11646. Code selection is based on location and
size. The lesion is on the right cheek, narrowing the range to 11640-11646. The largest diameter is 2.3 cm
plus .4 cm (2 mm + 2 mm on each side) making the excised diameter 2.7 cm. The correct code selection is
11643. Simple repair is not reported separately.

PTS: 1 DIF: Moderate


23. ANS: C
Rationale: In the CPT® Index, look for destruction, facial, and you are directed to code ranges 17000-17004,
17280-17286. 17000-17004 is for benign or pre-malignant lesions and 17280-17286 is for malignant lesions.
Actinic keratosis is a premalignant lesion so a code is chosen from code rage 17000-17004. Code selection is
based on the number of lesions destroyed. In this case, 19 lesions were destroyed making 17004 the correct
code choice. In ICD-9-CM, look under keratosis, actinic and you are directed to code 702.0. Verification of
the code in the Tabular List confirms code selection.

PTS: 1 DIF: Moderate


24. ANS: B
Rationale: Basal cell carcinoma is a malignant lesion. In the CPT® Index, look under Destruction, Skin,
Malignant and you are directed to code range 17260-17286, 96567. 96567 is for photodynamic therapy.
17260-17286 would be used for cryosurgery. Code selection is based on location and size. For the neck, a
code from range 17270-17276 would be selected. The neck lesion is 2.3 cm making 17273 the correct code.
For the cheek, a code from range 17280-17286 would be selected. The cheek lesion is 0.8 cm making 17281
the correct code choice. A modifier 51 is used on 17281 to indicate multiple surgeries.

PTS: 1 DIF: Moderate


25. ANS: A
Rationale: Rhomboid flap is a flap in the shape of a rhomboid used for a rotation flap. A rotation flap is
considered an adjacent tissue transfer. In the CPT® Index, look under Skin Graft and Flap, Tissue Transfer
and you are directed to code range 14000-14350. Code selection is based on location and flap size. The size
of the flap is calculated bin squire cm and includes both the size of the primary defect and secondary defect
created by the flap. The final measurements in this case are 2.7 cm x 2.1 cm which equals 5.67 cm 2. 14020
would be the correct code.

PTS: 1 DIF: Moderate


26. ANS: A
Rationale: Telangiectasias are small dilated blood vessels, commonly referred to as “spider veins,” or acne
rosacea—a benign lesion. Code 17106 is used to report the desctruction of cutaneous vascular proleferative
lesions. Because the total measurement in sq cm is not provided, you code to the lowest measurement.

PTS: 1 DIF: Moderate


27. ANS: C
Rationale: In the CPT® Index, look for Mastectomy, Radical and you are directed to code range 19303-
19306. CPT® code 19306 describes the Urban type procedure. A single pedicle TRAM flap is also
performed. TRAM is a transverse rectus abdominis myocutaneous flap method of breast reconstruction. For
the TRAM flap, look in the CPT® Index under TRAM Flap, Breast Reconstruction and you are directed to
code range 19367-19369. It can be performed with a double or a single pedicle flap. In this case, it is a
single flap with supercharging making 19368 the correct code choice. Modifier LT is used on both
procedures to indicate which side; and modifier 51 Multiple procedures is appended to the second procedure.

PTS: 1 DIF: Moderate


28. ANS: D
Rationale: Guidelines in the Adjacent Tissue Transfer or Rearrangement state that they are not to be used
when the repair of a laceration results in a configuration such as a Y plasty. Instructions in the guidelines for
repair state to add up all the lengths. Based on the documentation, the total length would be 18 cm. An
intermediate repair of this proportion would be described by code 12035.

PTS: 1 DIF: Moderate


29. ANS: D
Rationale: Based on the documentation, the total number of skin tags removed is 22. Code 11200 is reported
for the removal of “up to and including 15 lesions.” Notice the wording for 11201 “… each additional 10
lesions, or part thereof.” You do not need to use the modifier 52 to show less then 10 additional tags were
removed because of the code description. Code 11201 is an add-on code and modifier 51 Multiple procedures
exempt.

PTS: 1 DIF: Moderate


30. ANS: B
Rationale: In the CPT® Index, Advancement Flap directs you to see Skin, Adjacent Tissue Transfer, which
directs you to code range 14000-14350. Adjacent tissue transfer or rearrangement includes lesion excision
and is selected based on size and location. The defect is 11 cm x 5 cm (55 cm 2) and located on the knee. Due
to the size being 55 sq cm, the correct code is14301.

PTS: 1 DIF: Moderate


31. ANS: A
Rationale: The indication of extensive undermining of the wound and the use of multiple suture materials
supports the use of a complex closure for this patient; additionally the lesion of the neck is coded based on
size. After the lesion of the neck is removed the provider took two biopsies on the cheek supporting the use
of CPT® 11100 and the add-on code 11101 for the additional biopsy. It is necessary to use modifier -59 for
the biopsy because it is bundled into the excision and we need to indicate a separately identifiable procedure
performed at the same time. Additionally, because the pathology report has been returned showing the
biopsies were actinic keratosis, you would code accordingly.

PTS: 1 DIF: Difficult


32. ANS: D
Rationale: A Ying Yang flap is a rotation trap which is coded using Adjacent Tissue Transfer codes. In the
CPT® Index, look under Skin Graft and Flap, Tissue Transfer and you are directed to codes 14000-14350.
When the defect size is less than 30 sq cm, it is coded based on location and size. When it is more than 30 sq
cm, it is coded using 14301 and 14302. In this case, we have a flap that is 36.25 sq cm. 14301 is reported for
the first 30 sq cm – 60.0 sq cm. Wound preparation was also performed, so code 15004 should also be
reported.

PTS: 1 DIF: Difficult


33. ANS: A
Rationale: First it is important to separate the simple, intermediate and complex closures in the scenario.
Next it is necessary to look at each body area involved within each closure type. By adding the simple
closures of the right forearm of 7.6 cm and the simple closure of the right wrist of 6.5 cm you would code
12004 for the total of 14.10 cm. Next the intermediate closures are separated by area with the arm of 5.7 cm,
the upper chest of 10.3 cm and the right abdominal area of 8.9 cm are added together totally 24.90 cm for
CPT® 12036, leaving the cheek to be coded as 12052. Last we turn our attention to the complex closure of
the neck and ear, because they are in different body areas they are coded separately with CPT® 13152 for the
ear and 13132 for the neck.
PTS: 1 DIF: Difficult
34. ANS: A
Rationale: The excision of the lesion is found by looking in the CPT® Index under Excision, Lesion, Skin,
Malignant and you are directed to code range 11600-11646. The lesion is on the ankle (leg) narrowing the
code range to 11600-11606. The lesion is 2.4 cm making the correct code 11603. The guidelines for Excision
– Malignant Lesions tell us to report reconstructive closure (15002-15261, 15570-15770) separately. In this
case a split thickness skin graft was used directing us to code range 15100-15101, 15120-15121. 15100 is the
correct code choice. The diagnosis is squamous cell carcinoma. Look in the Neoplasm table for skin, ankle
and use the Primary column making 173.7 the correct code.

PTS: 1 DIF: Difficult


35. ANS: B
Rationale: Understanding a sebaceous cyst is benign, the lesion is coded based on size and location for
11401. The note also indicates the wound was closed in layers allowing for the intermediate closure 12031.
In the ICD-9-CM Index, look under Cyst, sebaceous; or Cyst, skin (sebaceous is a nonessential modifier).
Both options direct you to 706.2.

PTS: 1 DIF: Difficult


36. ANS: B
Rationale: The procedures performed were an excision of malignant lesion on the forehead (face) 6.6 mm
(0.6cm) (11641), excision of malignant lesion on the cheek (face) 1.3 cm (11642), Two biopsies on the face
(11100, 11101), complex closure of the check, 2.1 cm (13131), and an intermediate repair of the forehead, 1.0
cm (12051). Because the suspicious lesions were biopsied and were separate sites you would code 11100 with
modifier -59 and then the add-on code of 11101 for the second biopsy. The diagnoses were Basal Cell
Carcinoma, forehead (173.3); Basal Cell Carcinoma, right cheek (173.3); Compound nevus, left nose (alar
crease is on the nose) (216.3); and Epidermal Nevus, left forehead (216.3). Even if one diagnosis code
reports two separate lesions, it should only be reported once. The diagnosis should be 173.3, 216.3.

PTS: 1 DIF: Difficult


37. ANS: A
Rationale: The first excision is for a malignant neoplasm of the left leg measuring 2.5 cm (11603) and
repaired with a split thickness skin graft measuring 5.1 sq cm. The second excision is a surgical wound
preparation of an open wound to the right leg (15002) which was repaired with a split thickness autograft
measuring 3.2 sq cm. The split thickness autografts are added together for a total graft size of 8.3 sq cm
(15100). Since the original surgery on the right leg was four months ago, this surgery is outside of any global
period, so no additional modifier is needed. Modifier 51 should be used to indicate multiple procedures. The
primary reason for the surgery is the skin cancer on the leg so using your neoplasm table you would find
173.7. In reading the indications for surgery take note that the surgeon is discussing the fact that the original
skin graft did not take and now there is an open wound, leaving you to code for the complication 996.54
which describes “Mechanical complication due to graft of other tissue, not elsewhere classified” and then
because the documentation indicates the history of skin cancer adding the V10.83 to further support the
medical necessity of the complication.

PTS: 1 DIF: Difficult


38. ANS: C
Rationale: In the CPT®, look for Removal, Foreign Body, Subcutaneous Tissue and you are directed to code
range 10120-10121. The surgeon indicated in the note that they considered this incision and removal of
foreign body to be complicated leading us code 10121. In the ICD-9-CM, look for Foreign body,
subcutaneous tissue and you are directed to 729.6. There is no mention of granuloma of the skin making
709.4 incorrect. The instructions for 729.6 state to use an additional code from V90.01-V90.9 to identify the
foreign body. V90.10 indicates a retained metal fragment.

PTS: 1 DIF: Difficult


39. ANS: B
Rationale: The physician performed an incision and drainage (I & D) of a cyst on the chest. To find the code,
look in the CPT® Index for Incision and Drainage, Cyst, Skin and you are directed to code choices 10040,
10600-10061. 10040 is for acne surgery. 10600-10061is for I & D of a cyst. Only one cyst was drained
making 10060 the correct code. In the ICD-9-CM Index under Cyst, skin, code 706.2 is indicated.
Verification in the Tabular List confirms code selection.

PTS: 1 DIF: Difficult


40. ANS: A
Rationale: The provider performed a puncture aspiration of a seroma (clear body fluid built up where tissue
has been removed by surgery). In the CPT® Index, look under Seroma and your only option is incision and
drainage. This is not an incision and drainage, but a puncture aspiration. 10160 is for puncture aspiration.
Even though it does not specifically state “seroma” it would be the code to report. This is not a staged return
to the operative suite for the puncture aspiration of the seroma. Modifier -78 is used because the patient is
returning to the operative suite with a complication in the global period. For the diagnosis, it should be
reported as a post-operative complication. In the ICD-9-CM Index, Seroma indexes to 998.13. Verification
in the Tabular List confirms code selection.

PTS: 1 DIF: Difficult


41. ANS: D
Rationale: Rationale: In CPT®, Yin-Yang flap repair falls under Adjacent Tissue Transfer codes, based on
the measurement calculating to 9.28 sq cm (2.9 cm x 3.2 cm = 9.28 sq cm) and the location of the scalp the
correct CPT® code is 14020. For the ICD-9-CM, using the neoplasm table under skin, then scalp you are
directed to 173.4. Basosquamous cell carcinoma is clinically between a basal cell carcinoma and a squamous
cell carcinoma, but it is still a malignancy and would be coded accordingly.

PTS: 1 DIF: Difficult


42. ANS: B
Rationale: In the CPT® index, look up Lipectomy/Suction Assisted. You are referred to 15876-15879.
Review the codes to choose the appropriate service. There were three body areas where liposuction was
performed. Code 15877 covers the liposuction of the posterior iliac crest and abdomen. Code 15879 covers
the liposuction of the thighs

PTS: 1 DIF: Difficult


43. ANS: B
Rationale: The wound was prepped with sharp debridement supporting the use of 15004. Then a split
thickness graft was performed. The measurement applies to the recipient area, which is stated as 60 sq cm. A
split thickness autograft to the foot for the first 100 sq cm is coded with 15120. The operative note states,
“The homograft is taking quite nicely, the wounds appear to be fairly clean; he is ready for autografting,”
indicating this is a staged procedure and modifier 58 would be appended. In the ICD-9-CM Index, look for
Diabetic, ulcer, foot, which directs you to 250.8X [707.15]. The 5th digit 0 indicates it is Type II diabetes.
Although there are complications, it does not indicate it is uncontrolled. 707.15 is used for ulcer of the foot.
PTS: 1 DIF: Difficult
44. ANS: D
Rationale: In the CPT® Index, look for Mohs Micrographic Surgery and you are directed to code range
17311-17315. Code selection is based on location and stages. This operative note indicates the location is on
the face and only one stage is performed, making 17311 the correct code choice. According to the guidelines
for Mohs Micrographic Surgery, repairs should be coded separately. This is a complex repair on the eyelid
measuring 3.4 cm making 13152 the correct code choice. Modifier 51 is used to indicate multiple procedures.
In the ICD-9-CM, look in the Neoplasm table for skin, canthus (eye), primary. The code is 173.1

PTS: 1 DIF: Difficult


45. ANS: B
Rationale: With breast reduction surgery “reduction mammaplasty” we are lead to CPT® 19318, because this
is a unilateral code per CPT® we would need to append modifier -50. Additionally the operative report
indicates that because so much breast tissue was removed the surgeon had to create free nipple grafts to
recreate the nipple. In the CPT® look for reconstruction, breast, nipple and you are directed to code range
19350-19355. 19350 is the correct code. Normally within reduction mammaplasty the patient’s nipple is just
moved into place after the removal of the breast tissue. The Nipple reconstruction is normally bundled into
the reduction surgery making it necessary to append modifier -59.

PTS: 1 DIF: Difficult


46. ANS: D
Rationale: In the CPT® Index, look for Breast, Inplants, Insertion and you are directed to code range 19340-
19342. This is a delayed insertion making 19342 the correct code choice. For the ICD-9-CM look for
deformity, breast, congenital and you are directed to code 757.6.

PTS: 1 DIF: Difficult


47. ANS: B
Rationale: A rhomboid flap is a tissue transfer flap, coded from range 14000-14350. Because the surgeon
indicated that the rhomboid flap extended into the lower eyelid area, you would only code for the eyelid flap.
The final measurement of the flap is 3.51 sq cm directing us to CPT® 14060

PTS: 1 DIF: Difficult


48. ANS: A
Rationale: Documentation indicates that a localization wire was placed prior to the surgery by a radiologist.
We are asked to select the code for the surgeon’s services. Code 19125 describes an excision of a breast
lesion, which was identified preoperatively with a radiology marker.

PTS: 1 DIF: Difficult


49. ANS: A
Rationale: CPT® guidelines under Excision—Malignant Lesions state that closure other than simple can be
coded separately. Excision codes are based on location and size. The documented size is 1.0 cm with 1.1 cm
on all sides, making the total size with two margins to be a 3.2 cm excision on the forehead (11644). The
intermediate closure is also based on location and size; and reported with code 12052, with modifier 51
Multiple procedures.

PTS: 1 DIF: Difficult


50. ANS: C
Rationale: Mohs codes are selected based on location and number of stages, each including up to five blocks.
There is an add-on code for each additional block after the first five. Code 17311 is for the first stage and
17312 for the second stage, based on the documentation of the site: “forehead.” The second stage consisted of
six tissue blocks, reported with the add-on code 17315.

PTS: 1 DIF: Difficult


51. ANS: D
Rationale: In the CPT® Index, first look up Arthroscopy, Surgical/Shoulder. Here you are directed to see
codes 29606-29828. When you turn to these codes in the Surgical section to review them, it is code 29819
you would report. This represents an arthroscopy on the shoulder to remove loose bodies.

PTS: 1 DIF: Easy


52. ANS: A
Rationale: A tendon transplant is an important part of the Joplin procedure. The extensor tendon is
transplanted to the head of the metatarsal bone.

PTS: 1 DIF: Easy


53. ANS: D
Rationale: The acromion is an extension of the scapula that meets the clavicle at the shoulder to form the
acromioclavicular joint.

PTS: 1 DIF: Easy


54. ANS: C
Rationale: Hallux rigidus is an arthritic condition of the first metatarsophalangeal joint and one of the
treatments for it is a cheilectomy.

PTS: 1 DIF: Easy


55. ANS: C
Rationale: Segmental instrumentation of the spine is a procedure used to repair a spinal defect where the
fixation device is attached at each end and at least one other place in between.

PTS: 1 DIF: Easy


56. ANS: C
Rationale: A greenstick fracture is one where the bone does not break completely through, and does not
protrude through the skin. This would be considered a closed fracture. The treatment is open; however, the
treatment is not considered when coding the diagnosis.

PTS: 1 DIF: Easy


57. ANS: D
Rationale: The supraspinatus muscle is part of the rotator cuff, and the injury is listed under tear. Although it
is a shoulder injury, that definition is not specific enough, and neither is the sprain code.

PTS: 1 DIF: Easy


58. ANS: A
Rationale: In the ICD-9-CM alphabetic index, look under Sprain, lumbar and you are directed to 847.2.
Verification of the code in the Tabular List confirms the code selection.

PTS: 1 DIF: Easy


59. ANS: B
Rationale: Plantar fasciitis is a painful condition of the tendons and/or fascia of the feet. It can be found in
the ICD-9-CM alphabetic index under Fasciitis, plantar.

PTS: 1 DIF: Easy


60. ANS: D
Rationale: Documentation shows the patient had a fracture of his femoral shaft. The fracture was repaired
with open reduction and internal fixation using an intramedullary nail and interlocking screws. Selection of
codes depends on the fracture site and the method of treatment (closed, open, or percutaneous). The range of
codes can be found in the index under Fracture/Femur/Peritrochanteric/Intramedullary Implant Shaft.

PTS: 1 DIF: Moderate


61. ANS: B
Rationale: The exchange of strut is coded with 20697. There is a note that 20697 is not to be used in
combination with 20672 or 20696. 20697 can be found in the index under Fixation
(Device)/Application/External.

PTS: 1 DIF: Moderate


62. ANS: B
Rationale: From the CPT® index, look up Biopsy/muscle. You are referred to 20200-20206. The biopsy is
taken through the skin, or percutaneously. Although the biopsy is deep, it is performed percutaneously, which
is reported with 20206.

PTS: 1 DIF: Moderate


63. ANS: C
Rationale: In the CPT® index, look up Fracture/Nasal Bone/Closed Treatment. You are referred to 21310-
21320. Review codes to choose the appropriate service. 21315 is the correct code to report a displaced nasal
fracture that is manipulated with the forceps to realign the nasal bones.

PTS: 1 DIF: Moderate


64. ANS: B
Rationale: In the CPT® index, look up Removal/Fixation Device. You are referred to 20670-20680. Review
the codes to choose the appropriate service. 20680 is the correct code since a deep incision was made all the
way to the bone to locate the wire for removal. Modifier F4 is reported to indicate the finger the procedure is
performed on.

PTS: 1 DIF: Moderate


65. ANS: B
Rationale: In the CPT® index, look up Neck/Tumor/Excision. You are referred to 21552-21558. Review the
codes to choose the appropriate service. 21552 is the correct code to report the excision of a 5-cm mass where
the surgeon had to incise all the way through the subcutaneous tissue to remove the mass. There is no
indication of the mass being benign or malignant.

PTS: 1 DIF: Moderate


66. ANS: B
Rationale: In the CPT® index, look up Injection/Trigger Points/Three or More Muscles. You are referred to
20553. Review the code to verify accuracy. 20553 covers the three muscles (rhomboid major, rhomboid minor
and scapular muscles) that had a total of four (multiple) trigger point injections. Codes for trigger point
injections are determined by the number of muscles involved not the number of injections administered.

PTS: 1 DIF: Moderate


67. ANS: A
Rationale: In the CPT® index, look up Injection/Joint. You are referred to 20600-20610. Review the codes to
choose appropriate service. 20610 is the correct code since the injection was given in the trochanteric bursa
(hip) for drug therapy.

PTS: 1 DIF: Moderate


68. ANS: C
Rationale: In the CPT® index, look up Exploration/Neck/Penetrating Wound. You are referred to 20100.
Review the code to verify accuracy. 20100 is the correct code since the patient was sent to the operating room
for exploration of a gunshot (penetrating trauma) wound to identify damaged structures. This code includes
removal of foreign bodies, ligation, or coagulation of minor blood vessels, damaged tissue debrided and/or
repaired, and wound closure.

PTS: 1 DIF: Moderate


69. ANS: A
Rationale: In the CPT® book, look up Fracture/Humerus/Open Treatment. You are referred to 23615-23616.
Review the codes to choose appropriate service. 23616 is the correct code since the surgeon had to surgically
cut (dissection) near the site of the injury to get to the fracture. The fracture repair included a prosthetic
replacement (implant head) and the repair of the tuberosities. Modifier LT is used to indicate that the
procedure was performed on the left side.

PTS: 1 DIF: Moderate


70. ANS: B
Rationale: In the CPT® index, look up Finger/Abscess/Incision and Drainage. You are referred to 26010-
26011. Review the codes to choose the appropriate service. 20610 is the correct code since the physician did
not have to incise all the way to subcutaneous tissue along with debridement.

PTS: 1 DIF: Moderate


71. ANS: C
Rationale: In the CPT® index, look up Excision/Tumor/Pelvis. You are referred to 27043-27049, and 27059.
Review the codes to choose the appropriate service. 27048 is the correct code to report that the surgeon had to
incise deep into the fascia to remove the 3-cm mass. There was no indication of the mass being benign or
malignant.

PTS: 1 DIF: Moderate


72. ANS: D
Rationale: In the CPT® index, look up Arthroscopy/Surgical/Shoulder 29806-29828. The procedure
performed was a decompression of the subacromial space with partial acromioplasty, 29826. The report states
that the anterior lip of the acromion was resected. Also mentioned is the removal of spurs from the distal
clavical. The report does not state that a distal claviculectomy was performed; therefore, 29824 is not
reported. The debridement of the distal clavicle, 29822, is not reported because it is in the same area or
compartment as the subacromial decompression and is bundled with 29826.

PTS: 1 DIF: Moderate


73. ANS: D
Rationale: In the CPT® index, look up Arthroscopy/Surgical/Wrist 29843-29848. Code 29846 describes the
arthroscopic excision and/or repair of triangular fibrocartilage and/or joint debridement.

PTS: 1 DIF: Moderate


74. ANS: A
Rationale: This is not the repair of a fracture; it is a repair of a malunion. In the CPT® index, look up
Repair/Radius/Malunion or Nonunion, 25400, 25415. Code 25400 reports the repair of a malunion of the
radius. There is no mention of an autograft; therefore, 25405 is incorrect.

PTS: 1 DIF: Moderate


75. ANS: B
Rationale: This is a supracondylar fracture of the elbow repaired by percutaneous fixation. In the CPT®
index, look up Fracture/Humerus/Supracondylar/Percutaneous Fixation, 24538.

PTS: 1 DIF: Moderate


76. ANS: B
Rationale: In the CPT® index, look up Fracture/Fibula/Open Treatment 27784, 27792, 27814. 27784 reports
an open treatment of a proximal fibular fracture or shaft fracture. The correct code is 27792, Open treatment
of a distal fibular fracture which includes internal fixation.

PTS: 1 DIF: Moderate


77. ANS: B
Rationale: The report states that the extensor retinaculum of the first extensor compartment was incised.
Look in the CPT® index under Incision/Wrist/Tendon Sheath 25000-25001. Code 25000 shows de Quervains
disease in the description. Code 25001 refers to the flexor tendon and this involved the extensor tendon
making 25000 correct.

PTS: 1 DIF: Moderate


78. ANS: C
Rationale: Code 23075 reports the excision of a soft tissue mass (tumor), subcutaneous. The mass was
removed with deep, blunt dissection; however, there is no mention of the depth and you cannot assume that
the mass was subfascial because of the word “deep”. The measurement of the mass isn’t documented resulting
in the default to the smallest measurement of less than 3 cm for code 23075.

PTS: 1 DIF: Moderate


79. ANS: C
Rationale: Fracture codes are based on the location of the fracture and the treatment method. Documentation
describes a closed reduction or a transcondylar facture, with manipulation and application of skeletal traction.
This is described with code 24535. This can be found in the CPT® index under
Fracture/Humerus/Supracondylar/Closed Treatment.

PTS: 1 DIF: Moderate


80. ANS: B
Rationale: Anterior approach is used to perform several procedures on the cervical spine. The corpectomy has
the highest RVUs and is listed first. Code 63081 is the removal of one single cervical segment. Arthrodesis,
anterior interbody technique is coded with 22554. Plates are used for anterior instrumentation and placed over
a total of six segments (C2, C3, C4, C5, and C6), 22846. Modifier 51 is appended to 22554 to indicate
multiple procedures. The application of the titanium cages is described by add-on code 22851; and the
morselized allograft is described by 20930—both of which are modifier 51 exempt.

PTS: 1 DIF: Difficult


81. ANS: A
Rationale: The 4-cm mass was removed from the soft tissue of the shoulder. To access the mass, the provider
had to go through the proximal aspect of the teres minor (muscle). The mass was located distal to the inferior
glenohumeral ligament (IGHL). Masses that are removed from joint areas as opposed to masses removed
close to the skin require special knowledge and become more of an orthopedic concern due to the joint
involvement. Therefore, it is coded from codes within the orthopedic section. 23076 is used because was
carried down through the proximal aspect of the teres minor.

PTS: 1 DIF: Difficult


82. ANS: D
Rationale: In the CPT® index, look up Arthroscopy/Surgical/Knee. You are referred to 29871-29889. Review
the codes to choose appropriate service. 29881 is the correct code since the tear was in the “medial meniscus”
and a meniscectomy was performed. 29877 is also coded since another part of the knee, in the patellofemoral
compartment, was debrided with a shaver. The surgery had started out as a “diagnostic procedure,” but that
changed when the physician decided to perform surgical procedures on the knee, rather than only examining
the knee for diagnostic purposes. Modifier 59 is used to report that the procedures were performed in different
compartments of the knee.

PTS: 1 DIF: Difficult


83. ANS: A
Rationale: In the CPT® index, look up Elbow/Dislocation/Closed Treatment. You are referred to 24600-
24605, and 24640. Review the codes to choose appropriate service. 24640 is the correct code to report
treatment of a dislocated nursemaid’s elbow with manipulation. In the ICD-9-CM index, look up
Nursemaid’s/elbow. You are referred to 832.2. Review the code in the tabular section to verify accuracy.
According to ICD-9-CM guidelines: A dislocation not indicated as closed or open should be classified as
closed. 832.2 requires the fifth digit 3 to indicate a medial dislocation of the elbow. In the ICD-9-CM
alphabetic index to External Causes of Injury and Poisoning, Section 3, look up Pulling/injury/due to/sudden
strenuous movement. You are referred to code E927.0. Review the code in tabular section to verify accuracy.
Modifier 54 is used to report that the physician performed the surgical portion of the service. The patient is
referred to an orthopedist for follow-up care.

PTS: 1 DIF: Difficult


84. ANS: D
Rationale: In the CPT® index, look up Hematoma/Leg, Upper. You are referred to 27301. Verify the code for
accuracy. Modifier 78 is appended to 27301 to indicate that an unplanned procedure related to the initial
procedure was performed during the postoperative period. In the ICD-9-CM index, look up
Complications/surgical procedures/hematoma. You are referred to 998.12. Review the code in the tabular
section for accuracy.

PTS: 1 DIF: Difficult


85. ANS: D
Rationale: In the CPT® index, look up Fracture/Humerus/Epicondyle/Closed Treatment. You are referred to
code 24560-24565. Review the codes to choose the appropriate service. 24565 is the correct code to report an
epicondyle fracture that was manipulated (reduced) without surgically incising to perform the procedure. In
the CPT® book, look up Dislocation/Elbow/Closed Treatment. You are referred to 24600-24605, and 24640.
Review the codes to choose appropriate service. 24605 is the correct code since patient was put under general
anesthesia to perform the procedure. Modifier 54 is used to report the physician performed the surgical
portion. The patient is referred to an orthopedist for follow up care. Modifier 51 is used to report that multiple
procedures were performed.

PTS: 1 DIF: Difficult


86. ANS: C
Rationale: In the CPT® book, look up Trigger Finger Repair. You are referred to 26055. Review the code to
verify accuracy. In the CPT® book look up Injection/Joint. You are referred to 20600-20610. Review the
codes to choose appropriate service. 20610 is the correct code since the shoulder was injected. Modifier F6 is
used to report the right index finger that was repaired. Modifier LT is used to indicate the left shoulder joint.
Modifier 51 is used to indicate multiple procedures were performed.

PTS: 1 DIF: Difficult


87. ANS: A
Rationale: When reporting the removal of hardware (pins, screws, nails, rods), the code is selected by
fracture site, not the number of items removed or the number of incisions that are made. To report 20670 or
20680 more than once, there would need to be more than one fracture site involved. In this case, there is only
one fracture site even though two incisions are made. We know that the removal is deep because the screws
were in the bone. From the CPT® index, look up Removal/Implantation. The correct code is 20680.

PTS: 1 DIF: Difficult


88. ANS: C
Rationale: As noted in ICD-9-CM at the beginning of Chapter 17, a comminuted fracture is a closed fracture;
therefore, a comminuted lateral condyle fracture is listed as 812.42. The fracture code is found in the CPT®
index under Fracture/Humerus/Condyle/Open Treatment 24579. The manipulation is included in 24579. The
application of the first cast is always bundled with the 24579.

PTS: 1 DIF: Difficult


89. ANS: A
Rationale: This is the repair of a nonunion of a tibial fracture; therefore, you must look in the CPT® index
under Repair/Tibia, 27720-27725. The correct code is 27724, Repair of nonunion or malunion, tibia; with
iliac or other autograft (includes obtaining graft). This was a nonunion of a fracture, 733.82 which was a late
effect of a fracture of the lower extremity, 905.4.

PTS: 1 DIF: Difficult


90. ANS: B
Rationale: This was an open femoral shaft fracture, 821.11, thus eliminating C and D. The debridement prior
to the intramedullary rodding is reported with 11012 which is found in CPT® index under
Debridement/Bone/with Open Fracture and/or Dislocation which eliminates choice A.

PTS: 1 DIF: Difficult


91. ANS: C
Rationale: Two codes are required for the arthrogram; the injection procedure and the radiologic supervision
and interpretation. Look in the CPT® index under Arthrography/Shoulder and you will see 73040 and
Arthrography/Shoulder/Injection for 23350. The correct answer is choice C. Modifier 26 is required to
indicate the radiologic professional service.

PTS: 1 DIF: Difficult


92. ANS: A
Rationale: Look in your HCPCS book for Kenolog-40 in the Table of Drugs, J3301and then check the tabular
listing to verify. This eliminates choices B, C. and D. If you are coding without the answers, you would look
in the CPT® index under Injection/Carpal Tunnel/Therapeutic, 20526.

PTS: 1 DIF: Difficult


93. ANS: A
Rationale: Look in the CPT® index for Excision/Metatarsal for 28110-28114, 28122, 28140. Code 28122
reports a partial excision or sequestrectomy of metatarsal bone. The CPT® index for Lesion/Foot leads to
28080, 28090. Code 28090 reports the excision of the ganglion of the foot. Modifier 51 is appended to
indicate multiple procedures performed during the same session.

PTS: 1 DIF: Difficult


94. ANS: A
Rationale: The wrist arthroscopy was performed to provide stabilization. Look in the CPT® index for
Arthroscopy/Surgical/Wrist for guidance to 29843-29848. Check the tabular listing and 29847 reports
arthroscopy of the wrist with internal fixation for fracture or instability. Although several K-wires were
passed, 29847 is reported only once.

PTS: 1 DIF: Difficult


95. ANS: B
Rationale: The physician fused the talonavicular, the calcaneocuboid and subtalar joints. The code can be
found in the CPT® index under Arthrodesis/Talus/Triple 28715. Allograft was taken from the fibula for the
arthrodesis, 20902. Modifier 51 is required to indicate multiple procedures during the same session.

PTS: 1 DIF: Difficult


96. ANS: A
Rationale: This is a fracture of the olecranon process which is located at the upper end of the ulna. Look in
the CPT® index under Fracture/Ulna/Olecranon/Open Treatment 24685.

PTS: 1 DIF: Difficult


97. ANS: B
Rationale: This was a surgical arthroscopy of the knee reported with codes in the range of 29871-29889. The
medial meniscectomy is reported with 29881. Only one procedure can be reported per compartment. In this
case the debridement (29877) and the synovectomy (29875) were performed in the medial compartment;
therefore, they are not reported. Also note that 29875 is listed as a separate procedure; therefore, not reported.

PTS: 1 DIF: Difficult


98. ANS: C
Rationale: The anterior cruciate ligament repair can be found in the CPT® index under Cruciate
Ligament/Arthroscopic Repair 29888-29889. This was the anterior cruciate ligament, 29888. A medial
meniscectomy was also performed which is reported with 29881. Modifier -51 is required to report multiple
procedures performed during the same session. The patella tendon bone graft is included in the 29888
procedure. The notchplasty is also bundled as only one procedure can be reported per compartment
(patellofemoral).

PTS: 1 DIF: Difficult


99. ANS: A
Rationale: The procedure was a surgical arthroscopy of the knee reported with codes in the range of 29871-
29889. The medial meniscectomy is reported with 29881. The microfracture is reported with 29879. Only
one procedure can be reported for each compartment. The microfracture was performed in the medial and the
patellofemoral compartments. The microfracture procedure is more complex than the meniscectomy and it is
listed first. Modifier 59 is required for the meniscectomy to indicate a separate compartment. Also list 51 to
indicated multiple procedures performed during the same session. The debridement, 29877 is bundled with
the other procedures.

PTS: 1 DIF: Difficult


100. ANS: A
Rationale: The procedure performed was an arthroplasty of the knee found in the CPT® index under
Arthroplasty/Knee 27437-27443, 27446-27447. This was a total knee arthroplasty with patella resurfacing
reported with 27447.

PTS: 1 DIF: Difficult


101. ANS: B
Rationale: In the CPT® Index, look up Decortication, Lung, with Parietal Pleurectomy. This directs us to code
32320.

PTS: 1 DIF: Easy


102. ANS: A
Rationale: The spleen is the larges single mass of lymphatic tissue.

PTS: 1 DIF: Easy


103. ANS: C
Rationale: In the ICD-9-CM alphabetic index (volume 2), look up pneumothorax then spontaneous. This
directs us to code 512.8. When we check to confirm in the tabular listing (volume 1) we see that 512.8 is the
correct code for reporting a spontaneous pneumothorax.

PTS: 1 DIF: Easy


104. ANS: B
Rationale: CPT® index first look up Biopsy, Mediastinum, Needle which directs you to code 32405.

PTS: 1 DIF: Easy


105. ANS: D
Rationale: In the CPT® Index, look up Sinusotomy, Frontal Sinus, Nonobliterative and you are directed to
code range 31086-31087. Code selection is based on whether it is a brown incision or coronal incision. 31086
is the correct code.

PTS: 1 DIF: Easy


106. ANS: B
Rationale: In the CPT® Index, look up Intubation, Endotracheal Tube. This directs us to code 31500 which is
for an emergency endotracheal intubation.

PTS: 1 DIF: Easy


107. ANS: B
Rationale: A septum is a partition. The nasal septum separates the nostrils.

PTS: 1 DIF: Easy


108. ANS: D
Rationale: In the CPT® Index, look for Thoracotomy, for Post-op Complication. This directs us to code
32120. Since post-op hemorrhage is considered a complication, code 32120 is the correct code to report.

PTS: 1 DIF: Easy


109. ANS: B
Rationale: A penny in the nostril is considered a foreign object. In the CPT® Index, look up Removal, Foreign
Body, Nose and you are directed to code 30300. The description of the code confirms this is a removal that
would be performed in a doctor’s office.
PTS: 1 DIF: Easy
110. ANS: D
Rationale: RSV stands for respiratory syncytial virus. Look in the ICD-9-CM Alphabetic Index (volume 2)
under respiratory syncytial virus. This directs us to code 079.6. When we check this in the tabular listing
(volume 1), we see that code 079.6 is correct for reporting RSV.

PTS: 1 DIF: Easy


111. ANS: A
Rationale: Look in the ICD-9-CM Index under Pyopneumothorax, with fistula. The code is 510.0. You are
able to confirm that 510.0 is for empyema (pus in the pleural space) with fistula in the tabular section. “Pyo”
is the prefix for pus; pneumothorax is air in the pleural cavity. Not all words listed in the index are carried
over into the code description in the tabular section. 510.9 would be used if there was no mention of the
fistula. The 512 category is for pneumothorax with no pus in the pleural space.

PTS: 1 DIF: Moderate


112. ANS: A
Rationale: Since the type of asthma is not indicated, the correct code 493.9. The fifth digit, 2, is appropriate as
the documentation states it is with exacerbation. Wheezing and shortness of breath are signs and symptoms of
an exacerbation of asthma and not reported separately. According to the Official ICD-9-CM Guidelines (Sect
I. B.6), do not report signs and symptoms when a definitive diagnosis has been established.

PTS: 1 DIF: Moderate


113. ANS: C
Rationale: RSV is the acronym for “respiratory syncytial virus.” In the ICD-9-CM Index, look for
Bronchiolitis. Acute is a nonessential modifier. Under Bronchiolitis, there is a subterm for respiratory
syncytial virus which directs us to 466.11. Verification in the Tabular List confirms code selection.

PTS: 1 DIF: Moderate


114. ANS: D
Rationale: Endoscopy guidelines state that surgical thoracoscopy always includes a diagnostic thoracoscopy
and, therefore, would not be coded separately. In the CPT® Index, look for Thoracoscopy, Surgical, with
excision pericardial cyst, Tumor and/or Mass and you are directed to 32661.

PTS: 1 DIF: Moderate


115. ANS: C
Rationale: The procedure performed now is a secondary rhinoplasty. In the CPT® Index, look up Rhinoplasty,
Secondary which directs you to code range 30430-30450. Code selection is based on the reason for the repair
and the extensiveness of the repair. 30450 reports a secondary revision including osteotomies and nasal tip
work.

PTS: 1 DIF: Moderate


116. ANS: A
Rationale: In the ICD-9-CM Alphabetic Index (volume 2), look under Edema, lung (pulmonary),
Postoperative. This directs us to code 518.4. Next, we would report the fluid overload. Look in the Index
under Overload, Fluid and you are directed to 276.69. See the etiology/manifestation guidelines (Section
I.A.6). The E code represents the “postoperative” portion. In the E code Alphabetic Index (volume 2) we
first look under complication of medical or surgical procedure or treatment. Under this we would look at due
to misadventure since the edema is fluid overload associated with infusion. We then look at misadventure and
failure and in dosage of infusion, excess amount of fluid. This directs us to E873.0.
PTS: 1 DIF: Moderate
117. ANS: D
Rationale: COPD stands for Chronic Obstructive Pulomary Disease. In the ICD-9-CM Alphabetic Index
(volume 2) look under disease, lung (for pulmonary), obstructive, with bronchitis. Acute is coded with
491.22. Verification in the Tabular Index confirms code selection.

PTS: 1 DIF: Moderate


118. ANS: B
Rationale: In the CPT® Index, look under Stem Cell, Harvesting. This directs us to code range 38205-
38206. Code selection is based on whether it is allogenic (from a donor) or autologic (from the patient). This
is allogenic making 38205 the correct code choice.

PTS: 1 DIF: Moderate


119. ANS: C
Rationale: In the CPT® Index, look up Resection, Diaphram which directs us to code range 39560-
39561. Code selection depends on the type of repair. The repair is with primary sutures which is considered a
simple repair making 39560 the correct code choice.

PTS: 1 DIF: Moderate


120. ANS: D
Rationale: Epistaxis is the term for nasal hemorrhage. In the CPT® Index, look up Packing, Nasal
Hemorrhage and you are directed to code range 30901-30906. Code selection is determined by whether the
procedure is posterior or anterior. This is posterior and is subsequent making the correct code 30906.
Modifier 50 indicates this was done bilaterally.

PTS: 1 DIF: Moderate


121. ANS: C
Rationale: In the CPT® Index, look under Lymph Nodes, Biopsy and you are directed to a series of codes.
Turn to codes 38500 and 38510-38530. Code 38510 represents the deep cervical nodes and the 50 modifier
that they were excised bilaterally. Next, look to code 38525. This code is appropriate for the reporting of the
deep axillary nodes excised. The RT modifier to indicate these were taken only from the right side and the 51
modifier is to indicate multiple procedures performed at same session.

PTS: 1 DIF: Moderate


122. ANS: A
Rationale: The mediastinum extends from the sternum to the vertebral column and contains all the thoracic
viscera, except the lungs.

PTS: 1 DIF: Moderate


123. ANS: C
Rationale: In the CPT® Index look up Mediastinum, Exploration which directs us to codes 39000-39010.
Code selection is made based on the approached used. In this case, it is a transthoracic arpproach making
39010 the correct code.

PTS: 1 DIF: Moderate


124. ANS: D
Rationale: In the CPT® Index, look for Trachostomy, Emergency and you are directed to code range 31603-
31605. Code selection is based on the approach. In this case, the approach is transtracheal making 31603 the
correct code choice. Since the toy is a foreign body, look up in volume 2 of ICD-9-CM, Foreign Body,
swallowed and you are directed to 938. Verification in the Tabular List confirms code selection.
PTS: 1 DIF: Moderate
125. ANS: B
Rationale : Transbrachial biopsies would be performed via a bronchoscopy. In the CPT® Index, look up
Bronchoscopy, Biopsy and we see codes 31625-31629, 31632-31633. 31628 represents a transbrachial
biopsy of one lobe. A parenthetical statement under this code indicates we are to use code 31632 for any
additional lobes biopsied transbrachial. Another parenthetical statement under code 31632 indicates we would
not report 31632 even when multiple biopsies are taken in a lobe.

PTS: 1 DIF: Moderate


126. ANS: C
Rationale: In the CPT® Index, look up Pneumonectomy. By looking at codes 32440-32500 we see that code
32445 represents the extrapleural pneumonectomy. Next, in the CPT® Index, look under empymectomy
which directs us to code 32540. There is also a parenthetical statement under code 32540 instructing us to
report the correct lung removal code with 32540 if performed.

PTS: 1 DIF: Moderate


127. ANS: A
Rationale: In the CPT® Index, look up Excision, Polyp, Nose, Extensive which directs you to 30115. In the
code descriptor for 30115 is indicates polyps plural. Thus, we would not report 30115 multiple times.

PTS: 1 DIF: Moderate


128. ANS: D
Rationale: According to the CPT guidelines for coding of endoscopies, a surgical sinus endoscopy includes a
sinusotomy and diagnostic endoscopy. In the CPT® Index, look up Sinus/Sinsuses, Maxillarym Antrostomy
and you are directed to code range 31256-31267. We see code 31267 represents a surgical maxillary
antrostomy with maxillary tissue removal.

PTS: 1 DIF: Moderate


129. ANS: B
Rationale: There are three turbinates on each side of the nose: superior, middle and inferior. These
turbinates may become swollen and require surgery to restore airflow.

PTS: 1 DIF: Moderate


130. ANS: C
Rationale: The carina is located at the tracheal bifurcation. The tracheal bifurcation is the opening of the
bronchi as it splits into left and right.

PTS: 1 DIF: Moderate


131. ANS: A
Rationale: Septum repair is septoplasty. In the CPT® Index, Septoplasty directs you to 30520. Under the
code there is a parenthetical statement to use 30140 for submucous resection of the turbinates. Modifier 51 is
used to indicate multiple procedures.

PTS: 1 DIF: Difficult


132. ANS: B
Rationale: This procedure is performed endoscopically, where a small opening (window) is made in the
pericardial sac to facilitate drainage. Only an incision is made; nothing is excised. In the CPT® Indec, look
for Pericardial Sac, Drainage and you are directed to code 32659.
PTS: 1 DIF: Difficult
133. ANS: A
Rationale: As a referral, this patient becomes a new patient to the otolaryngologist. With a comprehensive
history, detailed exam and moderate MDM we are able to code and E/M of 99203. Modifier 25 is appended to
the E/M as this E/M is separately identifiable from the services performed. The nasal endoscopy and
laryngoscopy can both be performed via the nasal cavity thus we would report the more inclusive of the two
codes – the diagnostic laryngoscopy 31575. The otolaryngologist has diagnosed chronic pansinusitis code
473.8 and chronic laryngotracheitis represented by code 476.1. Code 476.1 encompasses the LPR and the
chronic laryngitis and tracheitis.

PTS: 1 DIF: Difficult


134. ANS: B
Rationale: Chemopleurodesis is represented by codes 32560-32562. Code 32560 is appropriate for the
described actions taken to instill the talc used to treat recurrent pneumothorax. Indexing for ICD-9-CM – look
in volume 2 under pneumothorax. This directs us to code 512.8 which represents acute or chronic
pneumothorax.

PTS: 1 DIF: Difficult


135. ANS: A
Rationale: Epistaxis is the term for nasal hemorrhage. In the CPT® Index look up Packing, Nasal
Hemorrhage which directs you to code range 30901-30906. 30903 represents anterior packing for an
uncontrolled or extensive nasal hemorrhage. Modifier 50 indicates this was done in both nares (bilaterally).
ICD-9-CM indexing is Epistaxis which leads us to code 784.7.

PTS: 1 DIF: Difficult


136. ANS: C
Rationale: According to the ICD-9-CM Official Coding Guidelines Section I.B.6, signs and symptoms are
reported when a related definitive diagnosis has not been established. Swine flu was suspected but not
established therefore we code the symptoms the patient presented with.

PTS: 1 DIF: Difficult


137. ANS: B
Rationale: Code 32552 represents the indwelling tunneled chest tube removal and code 32550 the insertion of
a new indwelling catheter/tube. For the ICD-9-CM, use the Neoplasm Table and look up lung, lower lobe. In
the malignant primary column we are directed to code 162.5. Verification in the Tabular List confirms code
selection.

PTS: 1 DIF: Difficult


138. ANS: B
Rationale: According to the CPT® guidelines for coding of endoscopies, a surgical sinus endoscopy includes
a sinusotomy and diagnostic endoscopy. Code 31255 represents a total ethmoidectomy and code 30520 the
septoplasty. The fracturing of the turbinates would be inclusive to the procedures and not reported separately.
Modifier 50 indicates these procedures were both performed bilaterally and modifier 51 is reported with code
30520 to indicate multiple procedures performed at same session, for maximum reimbursement.

PTS: 1 DIF: Difficult


139. ANS: D
Rationale: In the CPT® Index, look up biopsy, lung, needle. This directs us to use code 32405. Code 77002 is
the appropriate code for the fluoroscopic guidance as indicated by the parenthetical statement under code
32405 and by reviewing the code descriptor for 77002. We have a diagnosis of small cell carcinoma of the
lung which is code 162.3. The signs and symptoms are no longer codes since we do have this definitive
diagnosis (ICD-9-CM Guidelines Section I.B.6) Brain metastasis is suspected but not confirmed so it would
not be reported. The chemotherapy is planned but not performed so it would not be reported either.

PTS: 1 DIF: Difficult


140. ANS: B
Rationale: We are removing a segment of the lung. In the CPT® Index, look up removal, lung and then
single segment. This directs us to use code 32484. We have a confirmed diagnosis of apical lung cancer which
is code 162.8. We find this by looking in the neoplasm table under lung, apical. In the primary malignant
column we are directed to code 162.8.

PTS: 1 DIF: Difficult


141. ANS: A
Rationale: Since the marble is a foreign body, look in the CPT® Index for Removal, Foreign Body, Nose.
Here we are directed to use code 30300. For the ICD-9-CM code first look up in volume 2 of ICD-9-CM
foreign body, then entering through orifice then nose or nostril. This directs us to code 932. We check 932 in
volume 1 and find it to be the appropriate diagnosis code.

PTS: 1 DIF: Difficult


142. ANS: B
Rationale: In the CPT® Index, look up tracheostomy, surgical closure, without plastic repair. This directs us
to use code 31820. We have a diagnosis of coma (780.01), head injury, unspecified (959.01) and diabetes
(250.00). We will also report V55.0 for attention to artificial openings the closure of the tracheostomy.

PTS: 1 DIF: Difficult


143. ANS: D
Rationale: DOA means the individual is dead on arrival thus the lungs will be harvested from a cadaver donor.
The removal of the lungs by physician A will be reported with 32850 representing plural cadaver donor
pnemonectomies (lung removals). The insertion of the lungs reported with 32853-LT, RT and the backbench
preparation of both lungs (bilateral) by physician reported with 32856. Because each procedure is separately
reported by different physicians, modifier 51 is not required.

PTS: 1 DIF: Difficult


144. ANS: A
Rationale: In the CPT index look up laryngoscopy, then direct. When we look to these codes in the
Respiratory section 31515-31571 we find 31571 is appropriate for the injection into the vocal cords using an
operating microscope. Indexing for ICD-9-CM – look in volume 2 under spasm then larynx, laryngeal. We are
directed to code 478.75 which is accurate when checked in volume 1.

PTS: 1 DIF: Difficult


145. ANS: C
Rationale: In the CPT index look up laryngoscopy, then direct. When we look to these codes in the
Respiratory section 31515-31571 we find 31561 is appropriate for a direct operative laryngoscopy with
arytenoidectomy using an operating microscope. Indexing for ICD-9-CM – look in volume 2 under paralysis
then vocal cord, then bilateral (partial). We are directed to code 478.33 which is accurate when checked in
volume 1.
PTS: 1 DIF: Difficult
146. ANS: B
Rationale: In the CPT® Index, look up thoracostomy, tube. We are directed to use code 32551. We do not
have any documentation of the E/M performed so we would not code this. The ED physician would not be
performing the surgery for other injuries so we would not bundle the tube insertion into any of those
procedures.

PTS: 1 DIF: Difficult


147. ANS: D
Rationale: In the CPT® Index, look up sinusotomy, sphenoid. When we look to these codes in the Respiratory
section 3150-31051 we find 31051 is appropriate for the reporting of biopsies taken in the sphenoids as well
as removal of mucosa and polyps. Indexing for ICD-9-CM codes is sinusitis – sphenoidal 473.3 (this is for
chronic) and polyp – sinus sphenoidal is 471.8.

PTS: 1 DIF: Difficult


148. ANS: B
Rationale: An E/M visit cannot be coded for two reasons. 1) not enough documentation to assign an
appropriate E/M level and 2) we provided a procedure/treatment and we would code this. To code the
neublizer treatment look in the CPT index under inhalation (for breathing) treatment. This directs us to many
codes. If we check these codes we find 94640 accurately represents a neublizer treatment. A definitive
diagnosis of croup is given thus we code 478.75 for this and not the signs and symptoms the child presented
with (ICD9-CM Guidelines Section I.B.6)

PTS: 1 DIF: Difficult


149. ANS: B
Rationale: The correct CPT® code for an emergency criocothyroid tracheostomy is code 31605. Code 862.29
represents an injury to other specified intrathoracic organs without mention of open wound and the E code
tells us this was a motor vehicle accident and the patient was the passenger in the vehicle.

PTS: 1 DIF: Difficult


150. ANS: A
Rationale: The selection of the partial mastectomy code is based on whether the axillary lymph nodes are
removed. Lymphadenectomy was not necessary and the mastectomy is coded as 19301. A sentinel lymph
node biopsy was performed, which is the removal of one to three lymph nodes, and represented by 38525
Biopsy or excision of lymph node(s); open, deep axillary node(s). An injection procedure is performed to
determine whether cancer has spread to the lymph nodes, represented by 38792 Injection procedure; for
identification of sentinel node. Technetium 99 is a low level radioactive element. If the Geiger counter probe
detected radioactive element in the lymph nodes, it would be an indication that the cancer had spread and a
lymphadenectomy would be required. Modifier 51 is reported because multiple procedures were performed.

PTS: 1 DIF: Difficult


151. ANS: C
Rationale: An artery has three layers: an outer layer of tissue, a muscular middle, and an inner layer of
epithelial cells.

PTS: 1 DIF: Easy


152. ANS: B
Rationale: The conduction system contains pacemaker cells, nodes, the bundle of HIS, and the Purkinje
fibers.
PTS: 1 DIF: Easy
153. ANS: B
Rationale: The heart valves are made of flaps (cusps/leaflets) that open and close like one way swinging
doors which prevent the blood from flowing back.

PTS: 1 DIF: Easy


154. ANS: B
Rationale: The left main coronary artery branches into two slightly smaller arteries: the left anterior
descending coronary artery and the left circumflex coronary artery.

PTS: 1 DIF: Easy


155. ANS: C
Rationale: The heart is divided into right and left sides by a septum, which is a muscular wall.

PTS: 1 DIF: Easy


156. ANS: B
Rationale: The Category III code states “involving a visceral vessel”, like the renal artery. The directions
under 0079T state “Use 0079T in conjunction with 0078T”. Code 0078T reports the endovascular repair of
the abdominal aortic aneurysm and 0079T reports the visceral extension prosthesis.

PTS: 1 DIF: Easy


157. ANS: B
Rationale: The codes for the insertion of a tunneled central venous access device with a subcutaneous pump
is 36563. This can be found in the Index under Venous Access Device, Insertion, Central.

PTS: 1 DIF: Easy


158. ANS: A
Rationale: Because it is past eight weeks and the patient is still symptomatic, according to ICD-9-CM the
patient has chronic coronary insufficiency or ischemia. The notes in the tabular for 414.8 state “Any
condition classifiable to 410 specified as chronic, or presenting with symptoms after 8 weeks from date of
infarction.

PTS: 1 DIF: Easy


159. ANS: C
Rationale: Cardio = heart, megaly = enlargement

PTS: 1 DIF: Easy


160. ANS: B
Rationale: Angio = vessel, plasty = repair

PTS: 1 DIF: Easy


161. ANS: D
Rationale: Since this is a combo graft, codes 33517-33523 must be coded for the venous portion of the graft.
Also, this is a Redo more than one month after the original surgery, so the add-on code 33530 is appropriate.

PTS: 1 DIF: Moderate


162. ANS: C
Rationale: You must read the question carefully because this is a ruptured aortic aneurysm involving the
common iliac, not a ruptured aneurysm of the common iliac. The index shows 35103 under Aneurysm
Repair, Abdominal Aorta. Check the tabular and 35103 is correct. Code 35102 is a repair of an aneurysm
that has not ruptured.

PTS: 1 DIF: Moderate


163. ANS: D
Rationale: Code 78451 indicates a perfusion study that is qualitative or quantitative. There is no mention of
cardiac blood pooling imaging which eliminates choices a and b. Code 78453 reports a planar study and this
was a SPECT study, thus eliminating c.

PTS: 1 DIF: Moderate


164. ANS: A
Rationale: Only one primary intervention can be reported for the coronary arteries. The hierarchy is
angioplasty, followed by atherectomy, and finally stent placement. The primary procedure then is the stent
placement in the right coronary artery, 92980-RC. The additional code for the stent placement in the left
anterior descending is 92981-LD, and the additional code for the angioplasty of the left circumflex is 92984-
LC. Each is reported with the corresponding coronary artery modifiers.

PTS: 1 DIF: Moderate


165. ANS: A
Rationale: In the ICD-9 alphabetic index pseudoaneurysm directs the user to aneurysm.

PTS: 1 DIF: Moderate


166. ANS: D
Rationale: CPT® guidelines state, “when the battery of a pacemaker … is changed, it is actually the pulse
generator that is changed” It should be reported with one code for removal and another for replacement of the
battery or pulse generator.

PTS: 1 DIF: Moderate


167. ANS: C
Rationale: 33214 for conversion of single chamber system to a dual chamber includes removal of previously
placed pulse generator, testing of existing lead, insertion of new lead, and insertion of new pulse generator.

PTS: 1 DIF: Moderate


168. ANS: A
Rationale: According to ICD-9-CM Official Guidelines, a causal relationship is always assumed with
hypertension and chronic kidney disease. The notes in the tabular above category 403 state to report an
additional code for the stage of chronic kidney disease. This is stage III; therefore, 585.3 is also reported.

PTS: 1 DIF: Moderate


169. ANS: C
Rationale: The aorta is the trunk of the system, so it is a non-selective catheterization, 36200. 75650 is the
radiologic S&I code for the arch aortography. Modifier 26 is required to report the professional service.

PTS: 1 DIF: Moderate


170. ANS: D
Rationale: 36245 is reported 3 times for selective placement of the catheter into the three renal arteries. The
modifier RT and LT will prevent the third listing 36245 from being denied as a duplicate. 75724 reports the
bilateral renal angiography and 75774 is for the additional selective renal angiography. Modifier 26 is
required to report the professional service only.

PTS: 1 DIF: Moderate


171. ANS: A
Rationale: A causal connection with the heart failure and one is assumed with CKD, so a combined code
404.91 is needed. The type of heart failure and stage of CKD are also needed to complete the coding.
Reference the Hypertension Table and Cardiorenal, with, chronic kidney disease, stage I to stage IV, and heart
failure, 404.91. The notes for category 404 state to assign an additional code for the stage of chronic renal
failure and to assign an additional code from category 428 to identify the type of heart failure.

PTS: 1 DIF: Moderate


172. ANS: C
Rationale: Since leads were place on the right atrium and ventricle, it is a dual chamber system. Two codes
are necessary to report placement of an epicardial system. The notes under 33202 direct the coder to 33212
and 33213.

PTS: 1 DIF: Moderate


173. ANS: A
Rationale: Although the physician stated a comprehensive EP study was performed, the right ventricular
pacing and recording and left atrial pacing and recording from the coronary sinus or left atrium were not
done. The components must be billed separately.

PTS: 1 DIF: Moderate


174. ANS: D
Rationale: The removal of the dual chamber pacing cardioverter-defibrillator electrodes is reported with
33244. The insertion of the epicardial electrodes is report with 33202. The removal (33241) and the insertion
(33240) of the cardioverter-defibrillator generators are reported separately. Modifier 51 is necessary to show
multiple procedures performed during the same session.

PTS: 1 DIF: Moderate


175. ANS: C
Rationale: The physician performed both professional components of the stress test. Modifier 26 is not
required because these services are professional services.

PTS: 1 DIF: Moderate


176. ANS: B
Rationale: Since the catheter was repositioned and separate studies were performed, both the aortography and
the extremity angiography are reported. Modifier 26 reports the professional service.

PTS: 1 DIF: Moderate


177. ANS: C
Rationale: The catheter was selectively placed in the right and in the left pulmonary artery (36014 –RT and
36014-59-LT). The catheter on the right was further selectively positioned into the right lower lobe 36015-
RT. 36014-RT is taken away because you only code to the highest level in a family. Now the selective
catheterization codes are 36015-RT and 36014-59-LT. Modifier 59 indicates another vascular family. The
bilateral pulmonary angiography is reported with 75743. The additional study of the right lower lobe is
reported with 75774. Modifier 26 reports the professional services.
PTS: 1 DIF: Moderate
178. ANS: C
Rationale: The answer can be found in the Index by referencing Catheterization, Arterial, Cutdown, 36625.

PTS: 1 DIF: Moderate


179. ANS: B
Rationale: Age of patient is 5 years or older (36571). Fluoroscopic guidance for central venous access is
reported with 77001. Modifier 26 is necessary to show the professional service only.

PTS: 1 DIF: Moderate


180. ANS: B
Rationale: A central venous tunneled catheter with a subcutaneous access device with pump is reported with
36563. This can be found in the Index be referencing Venous Access Device, Insertion, Central.

PTS: 1 DIF: Moderate


181. ANS: B
Rationale: The left renal artery is a first order vessel (36245-LT). Angiography of the left renal vessel was
performed; however, there is no mention in the report of the results of the angiography. This is not a
diagnostic angiography, rather it is angiography for mapping (checking out known stenosis). The stent was
deployed (37205) in the left renal. The radiologic supervision and interpretation for the stent placement is
reported with 75960. Modifier 26 reports the professional service only. Follow up renal angiography is
bundled with the stent procedure.

PTS: 1 DIF: Difficult


182. ANS: B
Rationale: A mechanical valve was placed (33405). A one-artery, one-venous CABG was performed (33533,
33517). The left radial artery is an upper extremity artery and separately reportable (35600) as noted in the
guidelines preceding categories Combined Arterial Venous Grafting for Coronary Artery Bypass and
preceding Arterial Grafting for Coronary Artery Bypass.

PTS: 1 DIF: Difficult


183. ANS: B
Rationale: There are three parts to cardiac catheterization: selective catheter placement, injection of contrast,
and radiologic supervision and interpretation and report which are included in most of the cardiac
catheterization codes. Code 93460 includes right and left heart catheterization, coronary angiography, and left
ventriculography. None of the combined right and left heart catheterizations include right atrial angiography;
therefore, the add-on code 93565 is report. Modifier 26 is required to report the professional service.

PTS: 1 DIF: Difficult


184. ANS: C
Rationale: Moderate sedation is bundled, so it would not be reported. 35460 is for open procedure.
Venoplasty includes three zones for AV fistulas: the A/V graft and peripheral veins, the central veins, and the
vena cava. Only one venoplasty is reported for the A/V graft and peripheral veins. Modifier 26 reports the
professional services.

PTS: 1 DIF: Difficult


185. ANS: D
Rationale: CPT guidelines under Vascular Injection Procedures state the above listed in d as being included.
PTS: 1 DIF: Difficult
186. ANS: D
Rationale: This is a Category III code. Modifier 26 reports the professional service.

PTS: 1 DIF: Difficult


187. ANS: A
Rationale: 36015-RT reports the second order selective catheterization of the right pulmonary artery, 36014-
59-LT reports the first order selective catheterization in a different family of the left pulmonary artery, 75743-
26 reports bilateral pulmonary angiography, and 75774 reports the additional angiography after the basic
study of the right and left pulmonary arteries.

PTS: 1 DIF: Difficult


188. ANS: B
Rationale: The primary procedure was the placement of a stent in the RC. Angioplasty and AngioJet
atherectomy was unsuccessful in the LD, so stent was performed. Although multiple stents were placed only
one intervention can be reported per RC, LC or LD. The hierarchy of interventions is angioplasty, followed
by atherectomy, followed by stent placement. The stent placement supercedes the angioplasty and
atherectomy, therefore, only the additional stent placement is reported for the LD. A thrombectomy was
performed in the LD, 92973. A temporary pacemaker was inserted through the femoral vein; however, that is
bundled with the cardiac catheterization. At the end of the procedure an intra-aortic balloon pump was
inserted, 33967.

PTS: 1 DIF: Difficult


189. ANS: A
Rationale: Patient has a congenital cardiac anomaly, so 93303 is reported. Since it was performed in the
physician’s clinic, the global service is reported – meaning no modifier is necessary.

PTS: 1 DIF: Difficult


190. ANS: D
Rationale: According to CPT guidelines, codes 93293-93296 may be reported once every 90 days. Modifier
52 is not an acceptable modifier for 93293 and it is not necessary to report a reduced service since the
description of the codes states once up to 90 days.

PTS: 1 DIF: Difficult


191. ANS: C
Rationale: This is an exploration and modifier 78 is necessary because this is an unplanned return to the OR
by the same physician during the global period of another procedure. Modifier 78 is used for a return to the
OR for complications. This was an exploration for postoperative hemorrhage of the chest, 35820.

PTS: 1 DIF: Difficult


192. ANS: C
Rationale: The procedure described above is extensive according to CPT definition.

PTS: 1 DIF: Difficult


193. ANS: B
Rationale: 33426 reports mitral valve valvuloplasty with a prosthetic ring while 33405 reports an aortic valve
replacement with cardiopulmonary bypass. Modifier 51 is required on the second procedure to indicate
multiple procedures performed during the same setting.

PTS: 1 DIF: Difficult


194. ANS: A
Rationale: Procurement of the arterial conduit is bundled into 33535 and reported with modifier 80 for the
surgical assistant according to the guidelines. An add-on code, 35600, can be used for harvesting an artery of
the upper extremity; however, there is no mention of this in the report. The guidelines in the tabular above
33535 state to use modifier 80 when a surgical assistant performs an arterial graft procurement.

PTS: 1 DIF: Difficult


195. ANS: C
Rationale: The nonselective catheter placement in the aorta is reported with 36200. Contrast was injected
from one catheter placement site and there is a report for the aorta and the extremities making this an
abdominal aortogram with bilateral iliofemoral lower extremity angiography, 75630. Modifier 26 is required
for the professional service.

PTS: 1 DIF: Difficult


196. ANS: D
Rationale: 33235 reports the removal of the electrodes of a dual pacemaker lead system. Code 33208 reports
the replacement of permanent pacemaker generator with transvenous electrodes to the right atrium and right
ventricle and 33233 reports the removal of a pacemaker generator. Modifier 51 reports multiple procedures
performed during the same session.

PTS: 1 DIF: Difficult


197. ANS: C
Rationale: Code 93965 reports a complete bilateral noninvasive physiologic study of extremity veins. This
study can be found in the index by referencing Vascular Studies, Arterial Studies, Lower Extremity.

PTS: 1 DIF: Difficult


198. ANS: C
Rationale: The studies performed make up a comprehensive study (93619) which includes: evaluation with
right atrial pacing and recording, right ventricular pacing and recording, and His bundle recording without
induction of or attempted induction of arrhythmia. The left atrial pacing and recording (93621) and left
ventricular pacing and recording (93622) are add-on codes.

PTS: 1 DIF: Difficult


199. ANS: B
Rationale: Percutaneous balloon angioplasty (Maverick balloon used for dilatation) performed in the left
anterior descending coronary artery (LD). The cardiac catheterization is bundled into the interventional
procedure because there is no mention of a diagnostic cardiac catheterization. It appears that the diagnostic
cardiac catheterization was performed during another setting. The angioplasty, 92982 is reported with
modifier LD. Conscious sedation is included in the procedure.

PTS: 1 DIF: Difficult


200. ANS: D
Rationale: This was a combination arterial-venous graft (33517) and an arterial graft (33533). The upper
extremity artery graft procurement (35600) is separately reportable. 33517 and 35600 are add-on codes and
are modifier 51 exempt.

PTS: 1 DIF: Difficult


201. ANS: A
Rationale: Stent placement (92980) and IVUS (92978) are reportable. No 51 modifier on IVUS as it is an
add-on code. IVUS can be reported for each vessel when performed in multiple vessels; therefore, RC is
appended to 92978 to indicate the right coronary artery.

PTS: 1 DIF: Difficult

You might also like